OB Exam 3

Lakukan tugas rumah & ujian kamu dengan baik sekarang menggunakan Quizwiz!

The nurse is preparing to teach an antepartum client with GDM the correct method of administering an intermediate-acting insulin, such as neutral protamine Hagedorn (NPH), with a short-acting insulin (regular). In the correct order from 1 through 6, match the step number with the action needed to teach the client self-administration of this combination of insulin. a.Without adding air, withdraw the correct dose of NPH insulin. b.Gently rotate the insulin to mix it, and wipe the stopper. c.Inject air equal to the dose of NPH insulin into the vial, and remove the syringe. d.Inject air equal to the dose of regular insulin into the vial, and withdraw the medication. e.Check the insulin bottles for the expiration date. f.Wash hands. 1. Step 1 2. Step 2 3. Step 3 4. Step 4 5. Step 5 6. Step 6

1. ANS: F 2. ANS: E 3. ANS: B 4. ANS: C 5. ANS: D 6. ANS: A NOT: Regular insulin is always drawn up first when combining insulin. Other steps include ensuring that the insulin syringe corresponds to the concentration of insulin that is being used. The bottle should be checked before withdrawing the medication to be certain that it is the appropriate type.

1. Achieving and maintaining euglycemia are the primary goals of medical therapy for the pregnant woman with diabetes. These goals are achieved through a combination of diet, insulin, exercise, and blood glucose monitoring. The target blood glucose levels 1 hour after a meal should be _____________.

ANS: 110 to 129 mg/dl Target levels of blood glucose during pregnancy are lower than nonpregnant values. Accepted fasting levels are between 60 and 99 mg/dl, and 1-hour postmeal levels should be between 110 to 129 mg/dl. Two-hour postmeal levels should be 120 mg/dl or less.

18. The nurse caring for a newborn checks the record to note clinical findings that occurred before her shift. Which finding related to the renal system would be of increased significance and require further action? a. The pediatrician should be notified if the newborn has not voided in 24 hours. b. Breastfed infants will likely void more often during the first days after birth. c. Brick dust or blood on a diaper is always cause to notify the physician. d. Weight loss from fluid loss and other normal factors should be made up in 4 to 7 days.

ANS: A A newborn who has not voided in 24 hours may have any of a number of problems, some of which deserve the attention of the pediatrician. Formula-fed infants tend to void more frequently in the first 3 days; breastfed infants will void less during this time because the mother's breast milk has not yet come in. Brick dust may be uric acid crystals; blood spotting could be attributable to the withdrawal of maternal hormones (pseudomenstruation) or a circumcision. The physician must be notified only if the cause of bleeding is not apparent. Weight loss from fluid loss might take 14 days to regain.

12. When is a prophylactic cerclage for an incompetent cervix usually placed (in weeks of gestation)? a.12 to 14 b.6 to 8 c.23 to 24 d.After 24

ANS: A A prophylactic cerclage is usually placed at 12 to 14 weeks of gestation. The cerclage is electively removed when the woman reaches 37 weeks of gestation or when her labor begins. Six to 8 weeks of gestation is too early to place the cerclage. Cerclage placement is offered if the cervical length falls to less than 20 to 25 mm before 23 to 24 weeks. Although no consensus has been reached, 24 weeks is used as the upper gestational age limit for cerclage placement. DIF: Cognitive Level: Apply REF: p. 674 TOP: Nursing Process: Planning MSC: Client Needs: Health Promotion and Maintenance

7. A new mother states that her infant must be cold because the baby's hands and feet are blue. This common and temporary condition is called what? a.Acrocyanosis b.Erythema toxicum neonatorum c.Harlequin sign d.Vernix caseosa

ANS: A Acrocyanosis, or the appearance of slightly cyanotic hands and feet, is caused by vasomotor instability, capillary stasis, and a high hemoglobin level. Acrocyanosis is normal and intermittently appears over the first 7 to 10 days after childbirth. Erythema toxicum neonatorum (also called erythema neonatorum) is a transient newborn rash that resembles flea bites. The harlequin sign is a benign, transient color change in newborns. One half of the body is pale, and the other one half is ruddy or bluish-red with a line of demarcation. Vernix caseosa is a cheeselike, whitish substance that serves as a protective covering for the newborn.

18. What is the correct definition of a spontaneous termination of a pregnancy (abortion)? a.Pregnancy is less than 20 weeks. b.Fetus weighs less than 1000 g. c.Products of conception are passed intact. d.No evidence exists of intrauterine infection.

ANS: A An abortion is the termination of pregnancy before the age of viability (20 weeks). The weight of the fetus is not considered because some older fetuses may have a low birth weight. A spontaneous abortion may be complete or incomplete and may be caused by many problems, one being intrauterine infection. DIF: Cognitive Level: Remember REF: p. 669 TOP: Nursing Process: Assessment MSC: Client Needs: Health Promotion and Maintenance

31. Under which circumstance should the nurse immediately alert the pediatric provider? a. Infant is dusky and turns cyanotic when crying. b. Acrocyanosis is present 1 hour after childbirth. c. The infant's blood glucose level is 45 mg/dl. d. The infant goes into a deep sleep 1 hour after childbirth.

ANS: A An infant who is dusky and becomes cyanotic when crying is showing poor adaptation to extrauterine life. Acrocyanosis is an expected finding during the early neonatal life and is within the normal range for a newborn.Infants enter the period of deep sleep when they are approximately 1 hour old.

21. What is the highest priority nursing intervention when admitting a pregnant woman who has experienced a bleeding episode in late pregnancy? a.Assessing FHR and maternal vital signs b.Performing a venipuncture for hemoglobin and hematocrit levels c.Placing clean disposable pads to collect any drainage d.Monitoring uterine contractions

ANS: A Assessment of the FHR and maternal vital signs will assist the nurse in determining the degree of the blood loss and its effect on the mother and fetus. The most important assessment is to check the well-being of both the mother and the fetus. The blood levels can be obtained later. Assessing future bleeding is important it occurs, by definition, early in pregnancy. Placenta previa is a well-known reason for bleeding late in pregnancy. The premature separation of the placenta (abruptio placentae) is a bleeding disorder that can occur late in pregnancy. Cord insertion may cause a bleeding disorder that can also occur late in pregnancy. DIF: Cognitive Level: Understand REF: p. 669 TOP: Nursing Process: Assessment MSC: Client Needs: Physiologic Integrity, Physiologic Adaptation

28. Which intervention can nurses use to prevent evaporative heat loss in the newborn? a. Drying the baby after birth, and wrapping the baby in a dry blanket b. Keeping the baby out of drafts and away from air conditioners c. Placing the baby away from the outside walls and windows d. Warming the stethoscope and the nurse's hands before touching the baby

ANS: A Because the infant is wet with amniotic fluid and blood, heat loss by evaporation quickly occurs.Heat loss by convection occurs when drafts come from open doors and air currents created by people moving around. If the heat loss is caused by placing the baby near cold surfaces or equipment, it is referred to as a radiation heat loss. Conduction heat loss occurs when the baby comes in contact with cold surfaces.

12. A client is warm and asks for a fan in her room for her comfort. The nurse enters the room to assess the mother and her infant and finds the infant unwrapped in his crib with the fan blowing over him on high. The nurse instructs the mother that the fan should not be directed toward the newborn and that the newborn should be wrapped in a blanket. The mother asks why. How would the nurse respond? a. "Your baby may lose heat by convection, which means that he will lose heat from his body to the cooler ambient air. You should keep him wrapped, and should prevent cool air from blowing on him." b. "Your baby may lose heat by conduction, which means that he will lose heat from his body to the cooler ambient air. You should keep him wrapped, and should prevent cool air from blowing on him." c. "Your baby may lose heat by evaporation, which means that he will lose heat from his body to the cooler ambient air. You should keep him wrapped, and should prevent cool air from blowing on him." d. "Your baby will easily get cold stressed and needs to be bundled up at all times."

ANS: A Convection is the flow of heat from the body surface to cooler ambient air. Because of heat loss by convection, all newborns in open bassinets should be wrapped to protect them from the cold. Conduction is the loss of heat from the body surface to cooler surfaces, not air, in direct contact with the newborn. Evaporation is a loss of heat that occurs when a liquid is converted into a vapor. In the newborn, heat loss by evaporation occurs as a result of vaporization of moisture from the skin. Cold stress may occur from excessive heat loss; however, this does not imply that the infant will become stressed if not bundled at all times. Furthermore, excessive bundling may result in a rise in the infant's temperature.

22. Which order should the nurse expect for a client admitted with a threatened abortion? a.Bed rest b.Administration of ritodrine IV c.Nothing by mouth (nil per os [NPO]) d.Narcotic analgesia every 3 hours, as needed

ANS: A Decreasing the woman's activity level may alleviate the bleeding and allow the pregnancy to continue. Ritodrine is not the first drug of choice for tocolytic medications. Having the woman placed on NPO is unnecessary. At times, dehydration may produce contractions therefore, hydration is important. Narcotic analgesia will not decrease the contractions and may mask the severity of the contractions. DIF: Cognitive Level: Understand REF: pp. 671-672 TOP: Nursing Process: Planning MSC: Client Needs: Health Promotion and Maintenance

10. A mother is changing the diaper of her newborn son and notices that his scrotum appears large and swollen. The client is concerned. What is the best response from the nurse? a. "A large scrotum and swelling indicate a hydrocele, which is a common finding in male newborns." b. "I don't know, but I'm sure it is nothing." c. "Your baby might have testicular cancer." d. "Your baby's urine is backing up into his scrotum."

ANS: A Explaining what a hydrocele is and its characteristics is the most appropriate response by the nurse. The swelling usually decreases without intervention. Telling the mother that the condition is nothing important is inappropriate and does not address the mother's concern. Furthermore, if the nurse is unaware of any abnormal-appearing condition, then she should seek assistance from additional resources. Telling the mother that her newborn might have testicular cancer is inaccurate, inappropriate, and could cause the new mother undue worry. Urine will not back up into the scrotum if the infant has a hydrocele. Any nurse caring for the normal newborn should understand basic anatomy

3. A first-time father is changing the diaper of his 1-day-old daughter. He asks the nurse, "What is this black, sticky stuff in her diaper?" What is the nurse's best response? a. "That's meconium, which is your baby's first stool. It's normal." b. "That's transitional stool." c. "That means your baby is bleeding internally." d. "Oh, don't worry about that. It's okay."

ANS: A Explaining what meconium is and that it is normal is an accurate statement and the most appropriate response. Transitional stool is greenish-brown to yellowish-brown and usually appears by the third day after the initiation of feeding. Telling the father that the baby is internally bleeding is not an accurate statement. Telling the father not to worry is not appropriate. Such responses are belittling to the father and do not teach him about the normal stool patterns of his daughter.

2. Part of the health assessment of a newborn is observing the infant's breathing pattern. What is the predominate pattern of newborn's breathing? a.Abdominal with synchronous chest movements b.Chest breathing with nasal flaring c.Diaphragmatic with chest retraction d.Deep with a regular rhythm

ANS: A In a normal infant respiration, the chest and abdomen synchronously rise and infant breaths are shallow and irregular. Breathing with nasal flaring is a sign of respiratory distress. Diaphragmatic breathing with chest retraction is also a sign of respiratory distress.

16. Which explanation will assist the parents in their decision on whether they should circumcise their son? a. The circumcision procedure has pros and cons during the prenatal period. b. American Academy of Pediatrics (AAP) recommends that all male newborns be routinely circumcised. c. Circumcision is rarely painful, and any discomfort can be managed without medication. d. The infant will likely be alert and hungry shortly after the procedure.

ANS: A Parents need to make an informed choice regarding newborn circumcision, based on the most current evidence and recommendations. Health care providers and nurses who care for childbearing families should provide factual, unbiased information regarding circumcision and give parents opportunities to discuss the risks and benefits of the procedure. The AAP and other professional organizations note the benefits but stop short of recommending routine circumcision. Circumcision is painful and must be managed with environmental, nonpharmacologic, and pharmacologic measures. After the procedure, the infant may be fussy for several hours, or he may be sleepy and difficult to awaken for feeding.

21. A 3.8-kg infant was vaginally delivered at 39 weeks of gestation after a 30-minute second stage. A nuchal cord occurred. After the birth, the infant is noted to have petechiae over the face and upper back. Based on the nurse's knowledge, which information regarding petechiae should be shared with the parents? a. Petechiae (pinpoint hemorrhagic areas) are benign if they disappear within 48 hours of childbirth. b. These hemorrhagic areas may result from increased blood volume. c. Petechiae should always be further investigated. d. Petechiae usually occur with a forceps delivery.

ANS: A Petechiae that are acquired during birth may extend over the upper portion of the trunk and face. These lesions are benign if they disappear within 2 days of birth and no new lesions appear. Petechiae may result from decreased platelet formation. In this infant, the presence of petechiae is more likely a soft-tissue injury resulting from the nuchal cord at birth. Unless the lesions do not dissipate in 2 days, no reason exists to alarm the family. Petechiae usually occur with a breech presentation vaginal birth.

3. Screening at 24 weeks of gestation reveals that a pregnant woman has gestational diabetes mellitus (GDM). In planning her care, the nurse and the client mutually agree that an expected outcome is to prevent injury to the fetus as a result of GDM. This fetus is at the greatest risk for which condition? a.Macrosomia b.Congenital anomalies of the central nervous system c.Preterm birth d.Low birth weight

ANS: A Poor glycemic control later in pregnancy increases the rate of fetal macrosomia. Poor glycemic control during the preconception time frame and into the early weeks of the pregnancy is associated with congenital anomalies. Preterm labor or birth is more likely to occur with severe diabetes and is the greatest risk in women with pregestational diabetes. Increased weight, or macrosomia, is the greatest risk factor for this fetus.

14. In caring for the woman with DIC, which order should the nurse anticipate? a.Administration of blood b.Preparation of the client for invasive hemodynamic monitoring c.Restriction of intravascular fluids d.Administration of steroids

ANS: A Primary medical management in all cases of DIC involves a correction of the underlying cause, volume replacement, blood component therapy, optimization of oxygenation and perfusion status, and continued reassessment of laboratory parameters. Central monitoring would not be initially ordered in a client with DIC because it could contribute to more areas of bleeding. Management of DIC would include volume replacement, not volume restriction. Steroids are not indicated for the management of DIC. DIF: Cognitive Level: Apply REF: pp. 685-686 TOP: Nursing Process: Planning MSC: Client Needs: Physiologic Integrity

9. Which statement concerning the complication of maternal diabetes is the most accurate? a.Diabetic ketoacidosis (DKA) can lead to fetal death at any time during pregnancy. b.Hydramnios occurs approximately twice as often in diabetic pregnancies than in nondiabetic pregnancies. c.Infections occur about as often and are considered about as serious in both diabetic and nondiabetic pregnancies. d.Even mild-to-moderate hypoglycemic episodes can have significant effects on fetal well-being.

ANS: A Prompt treatment of DKA is necessary to save the fetus and the mother. Hydramnios occurs 10 times more often in diabetic pregnancies. Infections are more common and more serious in pregnant women with diabetes. Mild-to-moderate hypoglycemic episodes do not appear to have significant effects on fetal well-being.

2. Which condition would require prophylaxis to prevent subacute bacterial endocarditis (SBE) both antepartum and intrapartum? a.Valvular heart disease b.Congestive heart disease c.Arrhythmias d.Postmyocardial infarction

ANS: A Prophylaxis for intrapartum endocarditis and pulmonary infection may be provided for women who have mitral valve prolapse. Prophylaxis for intrapartum endocarditis is not indicated for a client with congestive heart disease, underlying arrhythmias, or postmyocardial infarction.

9. With regard to hemorrhagic complications that may occur during pregnancy, what information is most accurate? a.An incompetent cervix is usually not diagnosed until the woman has lost one or two pregnancies. b.Incidences of ectopic pregnancy are declining as a result of improved diagnostic techniques. c.One ectopic pregnancy does not affect a woman's fertility or her likelihood of having a normal pregnancy the next time. d.Gestational trophoblastic neoplasia (GTN) is one of the persistently incurable gynecologic malignancies.

ANS: A Short labors and recurring losses of pregnancy at progressively earlier gestational ages are characteristics of reduced cervical competence. Because diagnostic technology is improving, more ectopic pregnancies are being diagnosed. One ectopic pregnancy places the woman at increased risk for another one. Ectopic pregnancy is a leading cause of infertility. Once invariably fatal, GTN now is the most curable gynecologic malignancy. DIF: Cognitive Level: Understand REF: p. 675 TOP: Nursing Process: Assessment MSC: Client Needs: Health Promotion and Maintenance

5. The nurse is using the New Ballard Scale to determine the gestational age of a newborn. Which assessment finding is consistent with a gestational age of 40 weeks? a. Flexed posture b. Abundant lanugo c. Smooth, pink skin with visible veins d. Faint red marks on the soles of the feet

ANS: A Term infants typically have a flexed posture. Abundant lanugo; smooth, pink skin with visible veins; and faint red marks are usually observed on preterm infants.

35. Which newborn reflex is elicited by stroking the lateral sole of the infant's foot from the heel to the ball of the foot? a. Babinski b. Tonic neck c. Stepping d. Plantar grasp

ANS: A The Babinski reflex causes the toes to flare outward and the big toe to dorsiflex. The tonic neck reflex (also called the fencing reflex) refers to the posture assumed by newborns when in a supine position. The stepping reflex occurs when infants are held upright with their heel touching a solid surface and the infant appears to be walking. Plantar grasp reflex is similar to the palmar grasp reflex; when the area below the toes is touched, the infant's toes curl over the nurse's finger.

8. The nurse is preparing to administer a hepatitis B virus (HBV) vaccine to a newborn. Which intervention by the nurse is correct? a. Obtaining a syringe with a 25-gauge, 5/8-inch needle for medication administration b. Confirming that the newborn's mother has been infected with the HBV c. Assessing the dorsogluteal muscle as the preferred site for injection d. Confirming that the newborn is at least 24 hours old

ANS: A The HBV vaccine should be administered in the vastus lateralis muscle at childbirth with a 25-gauge, 5/8-inch needle and is recommended for all infants. If the infant is born to an infected mother who is a chronic HBV carrier, then the hepatitis vaccine and HBV immunoglobulin should be administered within 12 hours of childbirth.

25. The nurse should be cognizant of which important statement regarding care of the umbilical cord? a. The stump can become easily infected. b. If bleeding occurs from the vessels of the cord, then the nurse should immediately call for assistance. c. The cord clamp is removed at cord separation. d. The average cord separation time is 5 to 7 days.

ANS: A The cord stump is an excellent medium for bacterial growth. The nurse should first check the clamp (or tie) and apply a second one. If bleeding occurs and does not stop, then the nurse should call for assistance. The cord clamp is removed after 24 hours when it is dry. The average cord separation time is 10 to 14 days.

13. In caring for an immediate postpartum client, the nurse notes petechiae and oozing from her intravenous (IV) site. The client would be closely monitored for which clotting disorder? a.DIC b.Amniotic fluid embolism (AFE) c.Hemorrhage d.HELLP syndrome

ANS: A The diagnosis of DIC is made according to clinical findings and laboratory markers. A physical examination reveals unusual bleeding. Petechiae may appear around a blood pressure cuff on the woman's arm. Excessive bleeding may occur from the site of slight trauma such as venipuncture sites. These symptoms are not associated with AFE, nor is AFE a bleeding disorder. Hemorrhage occurs for a variety of reasons in the postpartum client. These symptoms are associated with DIC. Hemorrhage would be a finding associated with DIC and is not a clotting disorder in and of itself. HELLP syndrome is not a clotting disorder, but it may contribute to the clotting disorder DIC. DIF: Cognitive Level: Understand REF: p. 685 TOP: Nursing Process: Planning MSC: Client Needs: Physiologic Integrity

28. Nursing follow-up care often includes home visits for the new mother and her infant. Which information related to home visits is correct? a. Ideally, the visit is scheduled within 72 hours after discharge. b. Home visits are available in all areas. c. Visits are completed within a 30-minute time frame. d. Blood draws are not a part of the home visit.

ANS: A The home visit is ideally scheduled within 72 hours after discharge. This timing allows early assessment and intervention for problems with feedings, jaundice, newborn adaptation, and maternal-infant interaction. Because of geographic distances, home visits are not available in all locales. Visits are usually 60 to 90 minutes in length to allow enough time for assessment and teaching. When jaundice is found, the nurse can discuss the implications and check the transcutaneous bilirubin level or draw blood for testing.

14. Which major neonatal complication is carefully monitored after the birth of the infant of a diabetic mother? a.Hypoglycemia b.Hypercalcemia c.Hypobilirubinemia d.Hypoinsulinemia

ANS: A The neonate is at highest risk for hypoglycemia because fetal insulin production is accelerated during pregnancy to metabolize excessive glucose from the mother. At birth, the maternal glucose supply stops and the neonatal insulin exceeds the available glucose, thus leading to hypoglycemia. Hypocalcemia is associated with preterm birth, birth trauma, and asphyxia, all common problems of the infant of a diabetic mother. Excess erythrocytes are broken down after birth, and large amounts of bilirubin are released into the neonate's circulation, with resulting hyperbilirubinemia. Because fetal insulin production is accelerated during pregnancy, hyperinsulinemia develops in the neonate.

22. How would the nurse optimally reassure the parents of an infant who develops a cephalhematoma? a. A cephalhematoma may occur with a spontaneous vaginal birth. b. A cephalhematoma only happens as a result of a forceps- or vacuum-assisted delivery. c. It is present immediately after birth. d. The blood will gradually absorb over the first few months of life.

ANS: A The nurse should explain that bleeding between the skull and the periosteum of a newborn may occur during a spontaneous vaginal delivery as a result of the pressure against the maternal bony pelvis. Low forceps and other difficult extractions may result in bleeding. However, a cephalhematoma can also spontaneously occur. Swelling may appear unilaterally or bilaterally, is usually minimal or absent at birth, and increases over the first 2 to 3 days of life. Cephalhematomas gradually disappear over 2 to 3 weeks. A less common condition results in the calcification of the hematoma, which may persist for months.

19. What is the correct term for the cheeselike, white substance that fuses with the epidermis and serves as a protective coating? a. Vernix caseosa b. Surfactant c. Caput succedaneum d. Acrocyanosis

ANS: A The protection provided by vernix caseosa is needed because the infant's skin is so thin.Surfactant is a protein that lines the alveoli of the infant's lungs. Caput succedaneum is the swelling of the tissue over the presenting part of the fetal head. Acrocyanosis is cyanosis of the hands and feet, resulting in a blue coloring.

17. The most serious complication of an infant heelstick is necrotizing osteochondritis resulting from lancet penetration of the bone. What approach should the nurse take when performing the test to prevent this complication? a. Lancet should penetrate at the outer aspect of the heel. b. Lancet should penetrate the walking surface of the heel. c. Lancet should penetrate the ball of the foot. d. Lancet should penetrate the area just below the fifth toe.

ANS: A The stick should be made at the outer aspect of the heel and should penetrate no deeper than 2.4 mm. Repeated trauma to the walking surface of the heel can cause fibrosis and scarring that can lead to problems with walking later in life. The ball of the foot and the area below the fifth toe are inappropriate sites for a heelstick.

26. Which component of the sensory system is the least mature at birth? a. Vision b. Hearing c. Smell d. Taste

ANS: A The visual system continues to develop for the first 6 months after childbirth. As soon as the amniotic fluid drains from the ear (in minutes), the infant's hearing is similar to that of an adult. Newborns have a highly developed sense of smell and can distinguish and react to various tastes.

1. Which congenital anomalies can occur as a result of the use of antiepileptic drugs (AEDs) in pregnancy? (Select all that apply.) a.Cleft lip b.Congenital heart disease c.Neural tube defects d.Gastroschisis e.Diaphragmatic hernia

ANS: A, B, C Congenital anomalies that can occur with AEDs include cleft lip or palate, congenital heart disease, urogenital defects, and neural tube defects. Carbamazepine and valproate should be avoided if all possible; they may cause neural tube defects. Congenital anomalies of gastroschisis and diaphragmatic hernia are not associated with the use of AEDs.

3. Which statements regarding physiologic jaundice are accurate? (Select all that apply.) a. Neonatal jaundice is common; however, kernicterus is rare. b. Appearance of jaundice during the first 24 hours or beyond day 7 indicates a pathologic process. c. Because jaundice may not appear before discharge, parents need instruction on how to assess for jaundice and when to call for medical help. d. Jaundice is caused by reduced levels of serum bilirubin. e. Breastfed babies have a lower incidence of jaundice.

ANS: A, B, C Neonatal jaundice occurs in 60% of term newborns and in 80% of preterm infants. The complication called kernicterus is rare. Jaundice in the first 24 hours or that persists past day 7 is cause for medical concern. Parents need to be taught how to evaluate their infant for signs of jaundice. Jaundice is caused by elevated levels of serum bilirubin. Breastfeeding is associated with an increased incidence of jaundice.

MULTIPLE RESPONSE 1. Pain should be regularly assessed in all newborns. If the infant is displaying physiologic or behavioral cues that indicate pain, then measures should be taken to manage the pain. Which interventions are examples of nonpharmacologic pain management techniques? (Select all that apply.) a. Swaddling b. Nonnutritive sucking c. Skin-to-skin contact with the mother d. Sucrose e. Acetaminophen

ANS: A, B, C, D Swaddling, nonnutritive sucking, skin-to-skin contact with the mother, and sucrose are all appropriate nonpharmacologic techniques used to manage pain in neonates. Acetaminophen is a pharmacologic method of treating pain.

4. During life in utero, oxygenation of the fetus occurs through transplacental gas exchange. When birth occurs, four factors combine to stimulate the respiratory center in the medulla. The initiation of respiration then follows. What are these four essential factors? a. Chemical b. Mechanical c. Thermal d. Psychologic e. Sensory

ANS: A, B, C, E Chemical factors are essential to initiate breathing. During labor, decreased levels of oxygen and increased levels of carbon dioxide seem to have a cumulative effect that is involved in the initiation of breathing. Clamping of the cord may also contribute to the start of respirations and results in a drop in the level of prostaglandins, which are known to inhibit breathing. Mechanical factors are also necessary to initiate respirations. As the infant passes through the birth canal, the chest is compressed. After the birth, the chest is relaxed, which allows for negative intrathoracic pressure that encourages air to flow into the lungs. The profound change in temperature between intrauterine and extrauterine life stimulates receptors in the skin to communicate with the receptors in the medulla. The stimulation of these receptors also contributes to the initiation of breathing. Sensory factors include handling by the health care provider, drying by the nurse, lights, smells, and sounds. Psychologic factors do not contribute to the initiation of respirations.

2. Which statements describe the first stage of the neonatal transition period? (Select all that apply.) a. The neonatal transition period lasts no longer than 30 minutes. b. It is marked by spontaneous tremors, crying, and head movements. c. Passage of the meconium occurs during the neonatal transition period. d. This period may involve the infant suddenly and briefly sleeping. e. Audible grunting and nasal flaring may be present during this time.

ANS: A, B, C, E The first stage is an active phase during which the baby is alert; this stage is referred to as thefirst period of reactivity. Decreased activity and sleep mark the second stage, the period of decreased responsiveness. The first stage is the shortest, lasting less than 30 minutes. Such exploratory behaviors include spontaneous startle reactions. Audible grunting, nasal flaring, and chest retractions may be present; however, these behaviors usually resolve within 1 hour of life.

3. The reported incidence of ectopic pregnancy has steadily risen over the past 2 decades. Causes include the increase in sexually transmitted infections (STIs) accompanied by tubal infection and damage. The popularity of contraceptive devices such as the IUD has also increased the risk for ectopic pregnancy. The nurse suspects that a client has early signs of ectopic pregnancy. The nurse should be observing the client for which signs or symptoms? (Select all that apply.) a.Pelvic pain b.Abdominal pain c.Unanticipated heavy bleeding d.Vaginal spotting or light bleeding e.Missed period

ANS: A, B, D, E A missed period or spotting can be easily mistaken by the client as an early sign of pregnancy. More subtle signs depend on exactly where the implantation occurs. The nurse must be thorough in her assessment because pain is not a normal symptom of early pregnancy. As the fallopian tube tears open and the embryo is expelled, the client often exhibits severe pain accompanied by intraabdominal hemorrhage, which may progress to hypovolemic shock with minimal or even no external bleeding. In approximately one half of women, shoulder and neck pain results from irritation of the diaphragm from the hemorrhage. DIF: Cognitive Level: Apply REF: p. 676 TOP: Nursing Process: Assessment MSC: Client Needs: Physiologic Integrity

3. Diabetes refers to a group of metabolic diseases characterized by hyperglycemia resulting from defects in insulin action, insulin secretion, or both. Over time, diabetes causes significant changes in the microvascular and macrovascular circulations. What do these complications include? (Select all that apply.) a.Atherosclerosis b.Retinopathy c.Intrauterine fetal death (IUFD) d.Nephropathy e.Neuropathy f.Autonomic neuropathy

ANS: A, B, D, E These structural changes will most likely affect a variety of systems, including the heart, eyes, kidneys, and nerves. IUFD (stillbirth) remains a major complication of diabetes in pregnancy; however, this is a fetal complication.

3. The "Period of Purple Crying" is a program developed to educate new parents about infant crying and the dangers of shaking a baby. Each letter in the acronym "PURPLE" represents a key concept of this program. Which concepts are accurate? (Select all that apply.) a. P: peak of crying and painful expression b. U: unexpected c. R: baby is resting at last d. L: extremely loud e. E: evening

ANS: A, B, E P: peak of crying; U: unexpected—comes and goes; R: resists soothing; P: pain—line face; L: long—lasting up to 5 hours a day; and E: evening or late afternoon. Many hospitals now provide parents with an educational DVD and provide education before discharge.

1. A serious but uncommon complication of undiagnosed or partially treated hyperthyroidism is a thyroid storm, which may occur in response to stress such as infection, birth, or surgery. What are the signs and symptoms of this emergency disorder? (Select all that apply.) a.Fever b.Hypothermia c.Restlessness d.Bradycardia e.Hypertension

ANS: A, C Fever, restlessness, tachycardia, vomiting, hypotension, and stupor are symptoms of a thyroid storm. Fever, not hypothermia; tachycardia, not bradycardia; and hypotension, not hypertension, are symptoms of thyroid storm.

2. As recently as 2005, the AAP revised safe sleep practices to assist in the prevention of SIDS. The nurse should model these practices in the hospital and incorporate this information into the teaching plan for new parents. Which practices are ideal for role modeling? (Select all that apply.) a. Fully supine position for all sleep b. Side-sleeping position as an acceptable alternative c. "Tummy time" for play d. Infant sleep sacks or buntings e. Soft mattress

ANS: A, C, D The "back to sleep" position is now recommended as the only position for every sleep period. To prevent positional plagiocephaly (flattening of the head) the infant should spend time on his or her abdomen while awake and for play. Loose sheets and blankets may be dangerous because they could easily cover the baby's head. The parents should be instructed to tuck any bedding securely around the mattress or use sleep sacks or bunting bags instead. The side-sleeping position is no longer an acceptable alternative position, according to the AAP. Infants should always sleep on a firm surface, ideally a firm crib mattress covered by a sheet only. Quilts and sheepskins, among other bedding, should not be placed under the infant.

3. In caring for a pregnant woman with sickle cell anemia, the nurse must be aware of the signs and symptoms of a sickle cell crisis. What do these include? (Select all that apply.) a.Fever b.Endometritis c.Abdominal pain d.Joint pain e.Urinary tract infection (UTI)

ANS: A, C, D Women with sickle cell anemia have recurrent attacks (crises) of fever and pain, most often in the abdomen, joints, and extremities. These attacks are attributed to vascular occlusion when red blood cells (RBCs) assume the characteristic sickled shape. Crises are usually triggered by dehydration, hypoxia, or acidosis. Women with the sickle cell trait are usually at a greater risk for postpartum endometritis (uterine wall infection); however, this development is not likely to occur during the pregnancy and is not a sign for the disorder. Although women with sickle cell anemia are at an increased risk for UTIs, these infections are not an indication of a sickle cell crisis.

23. The nurse is circulating during a cesarean birth of a preterm infant. The obstetrician requests that cord clamping be delayed. What is the rationale for this directive? a. To reduce the risk for jaundice b. To reduce the risk of intraventricular hemorrhage c. To decrease total blood volume d. To improve the ability to fight infection

ANS: B Delayed cord clamping provides the greatest benefits to the preterm infant. These benefits include a significant reduction in intraventricular hemorrhage, a reduced need for a blood transfusion, and improved blood cell volume. The risk of jaundice can increase, requiring phototherapy. Although no difference in the newborn's infection fighting ability occurs, iron status is improved, which can provide benefits for 6 months.

2. Approximately 10% to 15% of all clinically recognized pregnancies end in miscarriage. What are possible causes of early miscarriage? (Select all that apply.) a.Chromosomal abnormalities b.Infections c.Endocrine imbalance d.Systemic disorders e.Varicella

ANS: A, C, D, E Infections are not a common cause of early miscarriage. At least 50% of pregnancy losses result from chromosomal abnormalities. Endocrine imbalances such as hypothyroidism or diabetes are also possible causes for early pregnancy loss. Other systemic disorders that may contribute to pregnancy loss include lupus and genetic conditions. Although infections are not a common cause of early miscarriage, varicella infection in the first trimester has been associated with pregnancy loss. DIF: Cognitive Level: Remember REF: p. 669 TOP: Nursing Process: Assessment MSC: Client Needs: Health Promotion and Maintenance

2. A lupus flare-up during pregnancy or early postpartum occurs in 15% to 60% of women with this disorder. Which conditions associated with systemic lupus erythematosus (SLE) are maternal risks? (Select all that apply.) a.Miscarriage b.Intrauterine growth restriction (IUGR) c.Nephritis d.Preeclampsia e.Cesarean birth

ANS: A, C, D, E Maternal risks associated with SLE include miscarriage, nephritis, preeclampsia, and cesarean birth. IUGR is a fetal risk related to SLE. Other fetal risks include stillbirth and prematurity.

4. Hearing loss is one of the genetic disorders included in the universal screening program. Auditory screening of all newborns within the first month of life is recommended by the AAP. What is the rationale for having this testing performed? (Select all that apply.) a. Prevents or reduces developmental delays b. Reassures concerned new parents c. Provides early identification and treatment d. Helps the child communicate better e. Is recommended by the Joint Committee on Infant Hearing

ANS: A, C, D, E New parents are often anxious regarding auditory screening and its impending results; however, parental anxiety is not the reason for performing the screening test. Auditory screening is usually performed before hospital discharge. Importantly, the nurse ensures the parents that the infant is receiving appropriate testing and fully explains the test to the parents. For infants who are referred for further testing and follow-up, providing further explanation and emotional support to the parents is an important responsibility for the nurse. All other responses are appropriate reasons for auditory screening of the newborn. Infants who do not pass the screening test should have it repeated. If the infant still does not pass the test, then he or she should have a full audiologic and medical evaluation by 3 months of age. If necessary, the infant should be enrolled in an early intervention program by 6 months of age.

1. A client who has undergone a D&C for early pregnancy loss is likely to be discharged the same day. The nurse must ensure that her vital signs are stable, that bleeding has been controlled, and that the woman has adequately recovered from the administration of anesthesia. To promote an optimal recovery, what information should discharge teaching include? (Select all that apply.) a.Iron supplementation b.Resumption of intercourse at 6 weeks postprocedure c.Referral to a support group, if necessary d.Expectation of heavy bleeding for at least 2 weeks e.Emphasizing the need for rest

ANS: A, C, E The woman should be advised to consume a diet high in iron and protein. For many women, iron supplementation also is necessary. The nurse should acknowledge that the client has experienced a loss, however early. She can be taught to expect mood swings and possibly depression. Referral to a support group, clergy, or professional counseling may be necessary. Discharge teaching should emphasize the need for rest. Nothing should be placed in the vagina for 2 weeks after the procedure, including tampons and vaginal intercourse. The purpose of this recommendation is to prevent infection. Should infection occur, antibiotics may be prescribed. The client should expect a scant, dark discharge for 1 to 2 weeks. Should heavy, profuse, or bright bleeding occur, she should be instructed to contact her health care provider. DIF: Cognitive Level: Apply REF: p. 672 TOP: Nursing Process: Implementation MSC: Client Needs: Physiologic Integrity

27. Which intervention by the nurse would reduce the risk of abduction of the newborn from the hospital? a. Instructing the mother not to give her infant to anyone except the one nurse assigned to her that day b. Applying an electronic and identification bracelet to the mother and the infant c. Carrying the infant when transporting him or her in the halls d. Restricting the amount of time infants are out of the nursery

ANS: B A measure taken by many facilities is to band both the mother and the baby with matching identification bracelets and band the infant with an electronic device that will sound an alarm if the infant is removed from the maternity unit. It is impossible for one nurse to be on call for one mother and baby for the entire shift; therefore, parents need to be able to identify the nurses who are working on the unit. Infants should always be transported in their bassinette for both safety and security reasons. All maternity unit nursing staff should have unique identification bracelets in comparison with the rest of the hospital. Infants should remain with their parents and spend as little time in the nursery as possible.

37. What is the rationale for evaluating the plantar crease within a few hours of birth? a. Newborn has to be footprinted. b. As the skin dries, the creases will become more prominent. c. Heel sticks may be required. d. Creases will be less prominent after 24 hours.

ANS: B As the infant's skin begins to dry, the creases will appear more prominent, and the infant's gestation could be misinterpreted. Footprinting nor heel sticks will not interfere with the creases. The creases will appear more prominent after 24 hours.

3. Which information should the nurse take into consideration when planning care for a postpartum client with cardiac disease? a.The plan of care for a postpartum client is the same as the plan for any pregnant woman. b.The plan of care includes rest, stool softeners, and monitoring of the effect of activity. c.The plan of care includes frequent ambulating, alternating with active range-of-motion exercises. d.The plan of care includes limiting visits with the infant to once per day.

ANS: B Bed rest may be ordered, with or without bathroom privileges. Bowel movements without stress or strain for the woman are promoted with stool softeners, diet, and fluids. Care of the woman with cardiac disease in the postpartum period is tailored to the woman's functional capacity. The woman will be on bed rest to conserve energy and to reduce the strain on the heart. Although the woman may need help caring for the infant, breastfeeding and infant visits are not contraindicated.

12. An 18-year-old client who has reached 16 weeks of gestation was recently diagnosed with pregestational diabetes. She attends her centering appointment accompanied by one of her girlfriends. This young woman appears more concerned about how her pregnancy will affect her social life than her recent diagnosis of diabetes. A number of nursing diagnoses are applicable to assist in planning adequate care. What is the most appropriate diagnosis at this time? a.Risk for injury, to the fetus related to birth trauma b.Deficient knowledge, related to diabetic pregnancy management c.Deficient knowledge, related to insulin administration d.Risk for injury, to the mother related to hypoglycemia or hyperglycemia

ANS: B Before a treatment plan is developed or goals for the outcome of care are outlined, this client must come to an understanding of diabetes and the potential effects on her pregnancy. She appears more concerned about changes to her social life than adopting a new self-care regimen. Risk for injury to the fetus related to either placental insufficiency or birth trauma may come later in the pregnancy. At this time, the client is having difficulty acknowledging the adjustments that she needs to make to her lifestyle to care for herself during pregnancy. The client may not yet be on insulin. Insulin requirements increase with gestation. The importance of glycemic control must be part of health teaching for this client. However, she has not yet acknowledged that changes to her lifestyle need to be made and may not participate in the plan of care until understanding takes place.

2. A perinatal nurse is giving discharge instructions to a woman, status postsuction, and curettage secondary to a hydatidiform mole. The woman asks why she must take oral contraceptives for the next 12 months. What is the bestresponse by the nurse? a."If you get pregnant within 1 year, the chance of a successful pregnancy is very small. Therefore, if you desire a future pregnancy, it would be better for you to use the most reliable method of contraception available." b."The major risk to you after a molar pregnancy is a type of cancer that can be diagnosed only by measuring the same hormone that your body produces during pregnancy. If you were to get pregnant, then it would make the diagnosis of this cancer more difficult." c."If you can avoid a pregnancy for the next year, the chance of developing a second molar pregnancy is rare. Therefore, to improve your chance of a successful pregnancy, not getting pregnant at this time is best." d."Oral contraceptives are the only form of birth control that will prevent a recurrence of a molar pregnancy."

ANS: B Beta-human chorionic gonadotropin (beta-hCG) hormone levels are drawn for 1 year to ensure that the mole is completely gone. The chance of developing choriocarcinoma after the development of a hydatidiform mole is increased. Therefore, the goal is to achieve a zero human chorionic gonadotropin (hCG) level. If the woman were to become pregnant, then it may obscure the presence of the potentially carcinogenic cells. Women should be instructed to use birth control for 1 year after treatment for a hydatidiform mole. The rationale for avoiding pregnancy for 1 year is to ensure that carcinogenic cells are not present. Any contraceptive method except an intrauterine device (IUD) is acceptable. DIF: Cognitive Level: Apply REF: p. 679 TOP: Nursing Process: Planning | Nursing Process: Implementation MSC: Client Needs: Physiologic Integrity

14. Which statement best describes the transition period between intrauterine and extrauterine existence for the newborn? a. Consists of four phases, two reactive and two of decreased responses b. Lasts from birth to day 28 of life c. Applies to full-term births only d. Varies by socioeconomic status and the mother's age

ANS: B Changes begin immediately after birth; the cutoff time when the transition is considered over (although the baby keeps changing) is 28 days. This transition period has three phases: first reactivity, decreased response, and second reactivity. All newborns experience this transition period, regardless of age or type of birth. Although stress can cause variations in the phases, the mother's age and wealth do not disturb the pattern.

5. A client with maternal phenylketonuria (PKU) has come to the obstetrical clinic to begin prenatal care. Why would this preexisting condition result in the need for closer monitoring during pregnancy? a.PKU is a recognized cause of preterm labor. b.The fetus may develop neurologic problems. c.A pregnant woman is more likely to die without strict dietary control. d.Women with PKU are usually mentally handicapped and should not reproduce.

ANS: B Children born to women with untreated PKU are more likely to be born with mental retardation, microcephaly, congenital heart disease, and low birth weight. Maternal PKU has no effect on labor. Women without dietary control of PKU are more likely to miscarry or bear a child with congenital anomalies. Screening for undiagnosed maternal PKU at the first prenatal visit may be warranted, especially in individuals with a family history of the disorder, with low intelligence of an uncertain cause, or who have given birth to microcephalic infants.

9. Which information regarding the care of antepartum women with cardiac conditions is mostimportant for the nurse to understand? a.Stress on the heart is greatest in the first trimester and the last 2 weeks before labor. b.Women with class II cardiac disease should avoid heavy exertion and any activity that causes even minor symptoms. c.Women with class III cardiac disease should get 8 to 10 hours of sleep every day and limit housework, shopping, and exercise. d.Women with class I cardiac disease need bed rest through most of the pregnancy and face the possibility of hospitalization near term.

ANS: B Class II cardiac disease is symptomatic with ordinary activity. Women in this category need to avoid heavy exertion and limit regular activities as symptoms dictate. Stress is greatest between weeks 28 and 32 of gestation, when hemodynamic changes reach their maximum. Class III cardiac disease is symptomatic with less-than-ordinary activity. These women need bed rest most of the day and face the possibility of hospitalization near term. Class I cardiac disease is asymptomatic at normal levels of activity. These women can perform limited normal activities with discretion, although they still need a good amount of sleep.

34. The process during which bilirubin is changed from a fat-soluble product to a water-soluble product is known as what? a. Enterohepatic circuit b. Conjugation of bilirubin c. Unconjugated bilirubin d. Albumin binding

ANS: B Conjugation of bilirubin is the process of changing the bilirubin from a fat-soluble to a water-soluble product and is the route by which part of the bile produced by the liver enters the intestine, is reabsorbed by the liver, and is then recycled into the intestine. Unconjugated bilirubin is a fat-soluble product. Albumin binding is the process during which something attaches to a protein molecule.

11. Which laboratory marker is indicative of DIC? a.Bleeding time of 10 minutes b.Presence of fibrin split products c.Thrombocytopenia d.Hypofibrinogenemia

ANS: B Degradation of fibrin leads to the accumulation of multiple fibrin clots throughout the body's vasculature. Bleeding time in DIC is normal. Low platelets may occur but are not indicative of DIC because they may be the result from other coagulopathies. Hypofibrinogenemia occurs with DIC. DIF: Cognitive Level: Remember REF: p. 684 TOP: Nursing Process: Assessment MSC: Client Needs: Physiologic Integrity

12. At 1 minute after birth a nurse assesses an infant and notes a heart rate of 80 beats per minute, some flexion of extremities, a weak cry, grimacing, and a pink body but blue extremities. Which Apgar score does the nurse calculate based upon these observations and signs? a. 4 b. 5 c. 6 d. 7

ANS: B Each of the five signs the nurse notes scores a 1 on the Apgar scale, for a total of 5. A score of 4 is too low for this infant. A score of 6 is too high for this infant. A score of 7 is too high for an infant with this presentation.

17. Which maternal condition always necessitates delivery by cesarean birth? a.Marginal placenta previa b.Complete placenta previa c.Ectopic pregnancy d.Eclampsia

ANS: B In complete placenta previa, the placenta completely covers the cervical os. A cesarean birth is the acceptable method of delivery. The risk of fetal death occurring is due to preterm birth. If the previa is marginal (i.e., 2 cm or greater away from the cervical os), then labor can be attempted. A cesarean birth is not indicated for an ectopic pregnancy. Labor can be safely induced if the eclampsia is under control. DIF: Cognitive Level: Understand REF: p. 681 TOP: Nursing Process: Assessment MSC: Client Needs: Physiologic Integrity

6. The nurse who is caring for a woman hospitalized for hyperemesis gravidarum would expect the initial treatment to involve what? a.Corticosteroids to reduce inflammation b.Intravenous (IV) therapy to correct fluid and electrolyte imbalances c.Antiemetic medication, such as pyridoxine, to control nausea and vomiting d.Enteral nutrition to correct nutritional deficits

ANS: B Initially, the woman who is unable to down clear liquids by mouth requires IV therapy to correct fluid and electrolyte imbalances. Corticosteroids have been successfully used to treat refractory hyperemesis gravidarum, but they are not the expected initial treatment for this disorder. Pyridoxine is vitamin B6, not an antiemetic medication. Promethazine, a common antiemetic, may be prescribed. In severe cases of hyperemesis gravidarum, enteral nutrition via a feeding tube may be necessary to correct maternal nutritional deprivation but is not the initial treatment for this client.

23. What is the nurse's initial action while caring for an infant with a slightly decreased temperature? a. Immediately notify the physician. b. Place a cap on the infant's head, and have the mother perform kangaroo care. c. Tell the mother that the infant must be kept in the nursery and observed for the next 4 hours. d. Change the formula; a decreased body temperature is a sign of formula intolerance.

ANS: B Keeping the head well covered with a cap prevents further heat loss from the head, and placing the infant skin-to-skin against the mother should increase the infant's temperature. Nursing actions are needed first to correct the problem. If the problem persists after the interventions, physician notification may then be necessary. A slightly decreased temperature can be treated in the mother's room, offering an excellent time for parent teaching on the prevention of cold stress. Mild temperature instability is an expected deviation from normal during the first days after childbirth as the infant adapts to external life.

19. What is the correct terminology for an abortion in which the fetus dies but is retained within the uterus? a.Inevitable abortion b.Missed abortion c.Incomplete abortion d.Threatened abortion

ANS: B Missed abortion refers to the retention of a dead fetus in the uterus. An inevitable abortion means that the cervix is dilating with the contractions. An incomplete abortion means that not all of the products of conception were expelled. With a threatened abortion, the woman has cramping and bleeding but no cervical dilation. DIF: Cognitive Level: Remember REF: p. 670 TOP: Nursing Process: Assessment MSC: Client Needs: Physiologic Integrity

1. A pregnant woman is being discharged from the hospital after the placement of a cervical cerclage because of a history of recurrent pregnancy loss, secondary to an incompetent cervix. Which information regarding postprocedural care should the nurse emphasize in the discharge teaching? a.Any vaginal discharge should be immediately reported to her health care provider. b.The presence of any contractions, rupture of membranes (ROM), or severe perineal pressure should be reported. c.The client will need to make arrangements for care at home, because her activity level will be restricted. d. The client will be scheduled for a cesarean birth

ANS: B Nursing care should stress the importance of monitoring for the signs and symptoms of preterm labor. Vaginal bleeding needs to be reported to her primary health care provider. Bed rest is an element of care. However, the woman may stand for periods of up to 90 minutes, which allows her the freedom to see her physician. Home uterine activity monitoring may be used to limit the woman's need for visits and to monitor her status safely at home. The cerclage can be removed at 37 weeks of gestation (to prepare for a vaginal birth), or a cesarean birth can be planned. DIF: Cognitive Level: Apply REF: p. 675 TOP: Nursing Process: Planning | Nursing Process: Implementation MSC: Client Needs: Health Promotion and Maintenance

11. A new mother with a thyroid disorder has come for a lactation follow-up appointment. Which thyroid disorder is a contraindication for breastfeeding? a.Hyperthyroidism b.PKU c.Hypothyroidism d.Thyroid storm

ANS: B PKU is a cause of mental retardation in infants; mothers with PKU pass on phenylalanine and therefore should elect not to breastfeed. A woman with either hyperthyroidism or hypothyroidism would have no particular reason not to breastfeed. A thyroid storm is a complication of hyperthyroidism and is not a contraindication to breastfeeding.

16. In contrast to placenta previa, what is the most prevalent clinical manifestation of abruptio placentae? a.Bleeding b.Intense abdominal pain c.Uterine activity d.Cramping

ANS: B Pain is absent with placenta previa and may be agonizing with abruptio placentae. Bleeding may be present in varying degrees for both placental conditions. Uterine activity and cramping may be present with both placental conditions. DIF: Cognitive Level: Understand REF: p. 683 TOP: Nursing Process: Diagnosis MSC: Client Needs: Physiologic Integrity

1. Preconception counseling is critical in the safe management of diabetic pregnancies. Which complication is commonly associated with poor glycemic control before and during early pregnancy? a.Frequent episodes of maternal hypoglycemia b.Congenital anomalies in the fetus c.Hydramnios d.Hyperemesis gravidarum

ANS: B Preconception counseling is particularly important since strict metabolic control before conception and in the early weeks of gestation is instrumental in decreasing the risk of congenital anomalies. Frequent episodes of maternal hypoglycemia may occur during the first trimester (not before conception) as a result of hormonal changes and the effects on insulin production and use. Hydramnios occurs approximately 10 times more often in diabetic pregnancies than in nondiabetic pregnancies. Typically, it is observed in the third trimester of pregnancy. Hyperemesis gravidarum may exacerbate hypoglycemic events because the decreased food intake by the mother and glucose transfer to the fetus contribute to hypoglycemia.

12. The client makes an appointment for preconception counseling. The woman has a known heart condition and is unsure if she should become pregnant. Which is the only cardiac condition that would cause concern? a.Marfan syndrome b.Eisenmenger syndrome c.Heart transplant d.Ventricular septal defect (VSD)

ANS: B Pregnancy is contraindicated in clients with Eisenmenger syndrome. Women who have had heart transplants are successfully having babies. However, conception should be postponed for at least 1 year after transplantation. Management of the client with Marfan syndrome during pregnancy includes bed rest, beta-blockers, and surgery before conception. VSD is usually corrected early in life and is therefore not a contraindication to pregnancy.

36. The condition during which infants are at an increased risk for subgaleal hemorrhage is called what? a.Infection b.Jaundice c.Caput succedaneum d.Erythema toxicum neonatorum

ANS: B Subgaleal hemorrhage is bleeding into the subgaleal compartment and is the result of the transition from a forceps or vacuum application. Because of the breakdown of the red blood cells within a hematoma, infants are at greater risk for jaundice. Subgaleal hemorrhage does not increase the risk for infections. Caput succedaneum is an edematous area on the head caused by pressure against the cervix. Erythema toxicum neonatorum is a benign rash of unknown cause that consists of blotchy red areas.

10. The parents of a newborn ask the nurse how much the newborn can see. The parents specifically want to know what type of visual stimuli they should provide for their newborn. What information provided by the nurse would be most useful to these new parents? a."Infants can see very little until approximately 3 months of age." b."Infants can track their parents' eyes and can distinguish patterns; they prefer complex patterns." c."The infant's eyes must be protected. Infants enjoy looking at brightly colored stripes." d."It's important to shield the newborn's eyes. Overhead lights help them see better."

ANS: B Telling the parents that infants can track their parents' eyes and can distinguish patterns is an accurate statement. Development of the visual system continues for the first 6 months of life. Visual acuity is difficult to determine, but the clearest visual distance for the newborn appears to be 19 cm. Infants prefer to look at complex patterns, regardless of the color. They prefer low illumination and withdraw from bright lights.

9. A primiparous woman is watching her newborn sleep. She wants him to wake up and respond to her. The mother asks the nurse how much he will sleep every day. What is an appropriate response by the nurse? a."He will only wake up to be fed, and you should not bother him between feedings." b."The newborn sleeps approximately 17 hours a day, with periods of wakefulness gradually increasing." c."He will probably follow your same sleep and wake patterns, and you can expect him to be awake soon." d."He is being stubborn by not waking up when you want him to. You should try to keep him awake during the daytime so that he will sleep through the night."

ANS: B Telling the woman that the newborn sleeps approximately 17 hours a day with periods of wakefulness that gradually increase is both accurate and the most appropriate response by the nurse. Periods of wakefulness are dictated by hunger, but the need for socializing also appears. Telling the woman that her infant is stubborn and should be kept awake during the daytime is an inappropriate nursing response.

16. Which information about variations in the infant's blood counts is important for the nurse to explain to the new parents? a. A somewhat lower-than-expected red blood cell count could be the result of a delay in clamping the umbilical cord. b. An early high white blood cell (WBC) count is normal at birth and should rapidly decrease. c. Platelet counts are higher in the newborn than in adults for the first few months. d. Even a modest vitamin K deficiency means a problem with the blood's ability to properly clot.

ANS: B The WBC count is normally high on the first day of birth and then rapidly declines. Delayed cord clamping results in an increase in hemoglobin and the red blood cell count. The platelet count is essentially the same for newborns and adults. Clotting is sufficient to prevent hemorrhage unless the deficiency of vitamin K is significant.

15. Bell palsy is an acute idiopathic facial paralysis, the cause for which remains unknown. Which statement regarding this condition is correct? a.Bell palsy is the sudden development of bilateral facial weakness. b.Women with Bell palsy have an increased risk for hypertension. c.Pregnant women are affected twice as often as nonpregnant women. d.Bell palsy occurs most frequently in the first trimester.

ANS: B The clinical manifestations of Bell palsy include the development of unilateral facial weakness, pain surrounding the ears, difficulty closing the eye, and hyperacusis. The cause is unknown; however, Bell palsy may be related to a viral infection. Pregnant women are affected at a rate of three to five times that of nonpregnant women. The incidence rate peaks during the third trimester and puerperium. Women who develop Bell palsy in pregnancy have an increased risk for hypertension.

1. A woman gave birth to a healthy 7-pound, 13-ounce infant girl. The nurse suggests that the client place the infant to her breast within 15 minutes after birth. The nurse is aware that the initiation of breastfeeding is most effective during the first 30 minutes after birth. What is the correct term for this phase of alertness? a.Transition period b.First period of reactivity c.Organizational stage d.Second period of reactivity

ANS: B The first period of reactivity is the first phase of transition and lasts up to 30 minutes after birth. The infant is highly alert during this phase. The transition period is the phase between intrauterine and extrauterine existence. An organizational stage is not a valid stage. The second period of reactivity occurs approximately between 4 and 8 hours after birth, after a period of sleep.

14. The nurse is completing a physical examination of the newborn 24 hours after birth. Which component of the evaluation is correct? a. The parents are excused to reduce their normal anxiety. b. The nurse can gauge the neonate's maturity level by assessing his or her general appearance. c. Once often neglected, blood pressure is now routinely checked. d. When the nurse listens to the neonate's heart, the S1 and S2 sounds can be heard; the S1sound is somewhat higher in pitch and sharper than the S2 sound.

ANS: B The nurse is looking at skin color, alertness, cry, head size, and other features. The parents' presence actively involves them in child care and gives the nurse the chance to observe their interactions. Blood pressure is not usually taken unless cardiac problems are suspected. The S2sound is higher and sharper than the S1 sound.

2. A new father wants to know what medication was put into his infant's eyes and why it is needed. How does the nurse explain the purpose of the erythromycin (Ilotycin) ophthalmic ointment? a. Erythromycin (Ilotycin) ophthalmic ointment destroys an infectious exudate caused byStaphylococcus that could make the infant blind. b. This ophthalmic ointment prevents gonorrheal and chlamydial infection of the infant's eyes, potentially acquired from the birth canal. c. Erythromycin (Ilotycin) prevents potentially harmful exudate from invading the tear ducts of the infant's eyes, leading to dry eyes. d. This ointment prevents the infant's eyelids from sticking together and helps the infant see.

ANS: B The nurse should explain that prophylactic erythromycin ophthalmic ointment is instilled in the eyes of all neonates to prevent gonorrheal and chlamydial infection that potentially could have been acquired from the birth canal. This prophylactic ophthalmic ointment is not instilled to prevent dry eyes and has no bearing on vision other than to protect against infection that may lead to vision problems.

4. A 26-year-old pregnant woman, gravida 2, para 1-0-0-1, is 28 weeks pregnant when she experiences bright red, painless vaginal bleeding. On her arrival at the hospital, which diagnostic procedure will the client most likely have performed? a.Amniocentesis for fetal lung maturity b.Transvaginal ultrasound for placental location c.Contraction stress test (CST) d.Internal fetal monitoring

ANS: B The presence of painless bleeding should always alert the health care team to the possibility of placenta previa, which can be confirmed through ultrasonography. Amniocentesis is not performed on a woman who is experiencing bleeding. In the event of an imminent delivery, the fetus is presumed to have immature lungs at this gestational age, and the mother is given corticosteroids to aid in fetal lung maturity. A CST is not performed at a preterm gestational age. Furthermore, bleeding is a contraindication to a CST. Internal fetal monitoring is also contraindicated in the presence of bleeding. DIF: Cognitive Level: Apply REF: p. 680 TOP: Nursing Process: Assessment MSC: Client Needs: Health Promotion and Maintenance

4. A 26-year-old primigravida has come to the clinic for her regular prenatal visit at 12 weeks. She appears thin and somewhat nervous. She reports that she eats a well-balanced diet, although her weight is 5 pounds less than it was at her last visit. The results of laboratory studies confirm that she has a hyperthyroid condition. Based on the available data, the nurse formulates a plan of care. Which nursing diagnosis is most appropriate for the client at this time? a.Deficient fluid volume b.Imbalanced nutrition: less than body requirements c.Imbalanced nutrition: more than body requirements d.Disturbed sleep pattern

ANS: B This client's clinical cues include weight loss, which supports a nursing diagnosis of "Imbalanced nutrition: less than body requirements." No clinical signs or symptoms support a nursing diagnosis of deficient fluid volume. This client reports weight loss, not weight gain. Although the client reports nervousness, the most appropriate nursing diagnosis, based on the client's other clinical symptoms, is "Imbalanced nutrition: less than body requirements."

18. It is extremely rare for a woman to die in childbirth; however, it can happen. In the United States, the annual occurrence of maternal death is 12 per 100,000 cases of live birth. What are the leading causes of maternal death? a.Embolism and preeclampsia b.Trauma and motor vehicle accidents (MVAs) c.Hemorrhage and infection d.Underlying chronic conditions

ANS: B Trauma is the leading cause of obstetric death in women of childbearing age. Most maternal injuries are the result of MVAs and falls. Although preeclampsia and embolism are significant contributors to perinatal morbidity, these are not the leading cause of maternal mortality. Maternal death caused by trauma may occur as the result of hemorrhagic shock or abruptio placentae. In these cases, the hemorrhage is the result of trauma, not childbirth. The wish to become a parent is not eliminated by a chronic health problem, and many women each year risk their lives to have a baby. Because of advanced pediatric care, many women are surviving childhood illnesses and reaching adulthood with chronic health problems such as cystic fibrosis, diabetes, and pulmonary disorders.

7. In terms of the incidence and classification of diabetes, which information should the nurse keep in mind when evaluating clients during their ongoing prenatal appointments? a.Type 1 diabetes is most common. b.Type 2 diabetes often goes undiagnosed. c.GDM means that the woman will receive insulin treatment until 6 weeks after birth. d.Type 1 diabetes may become type 2 during pregnancy.

ANS: B Type 2 diabetes often goes undiagnosed because hyperglycemia gradually develops and is often not severe. Type 2, sometimes called adult-onset diabetes, is the most common type of diabetes. GDM refers to any degree of glucose intolerance first recognized during pregnancy; insulin may or may not be needed. People do not go back and forth between type 1 and type 2 diabetes.

5. A laboring woman with no known risk factors suddenly experiences spontaneous ROM. The fluid consists of bright red blood. Her contractions are consistent with her current stage of labor. No change in uterine resting tone has occurred. The fetal heart rate (FHR) begins to decline rapidly after the ROM. The nurse should suspect the possibility of what condition? a.Placenta previa b.Vasa previa c.Severe abruptio placentae d.Disseminated intravascular coagulation (DIC)

ANS: B Vasa previa is the result of a velamentous insertion of the umbilical cord. The umbilical vessels are not surrounded by Wharton jelly and have no supportive tissue. The umbilical blood vessels thus are at risk for laceration at any time, but laceration occurs most frequently during ROM. The sudden appearance of bright red blood at the time of ROM and a sudden change in the FHR without other known risk factors should immediately alert the nurse to the possibility of vasa previa. The presence of placenta previa most likely would be ascertained before labor and is considered a risk factor for this pregnancy. In addition, if the woman had a placenta previa, it is unlikely that she would be allowed to pursue labor and a vaginal birth. With the presence of severe abruptio placentae, the uterine tonicity typically is tetanus (i.e., a boardlike uterus). DIC is a pathologic form of diffuse clotting that consumes large amounts of clotting factors, causing widespread external bleeding, internal bleeding, or both. DIC is always a secondary diagnosis, often associated with obstetric risk factors such as the hemolysis, elevated liver enzyme levels, and low platelet levels (HELLP) syndrome. This woman did not have any prior risk factors. DIF: Cognitive Level: Analyze REF: p. 684 TOP: Nursing Process: Diagnosis MSC: Client Needs: Physiologic Integrity

30. Which cardiovascular changes cause the foramen ovale to close at birth? a. Increased pressure in the right atrium b. Increased pressure in the left atrium c. Decreased blood flow to the left ventricle d. Changes in the hepatic blood flow

ANS: B With the increase in the blood flow to the left atrium from the lungs, the pressure is increased, and the foramen ovale is functionally closed. The pressure in the right atrium decreases at birth and is higher during fetal life. Blood flow increases to the left ventricle after birth. The hepatic blood flow changes but is not the reason for the closure of the foramen ovale.

1. What are the various modes of heat loss in the newborn? (Select all that apply.) a. Perspiration b. Convection c. Radiation d. Conduction e. Urination

ANS: B, C, D Convection, radiation, evaporation, and conduction are the four modes of heat loss in the newborn.

4. Autoimmune disorders often occur during pregnancy because a large percentage of women with an autoimmune disorder are of childbearing age. Which disorders fall into the category of collagen vascular disease? (Select all that apply.) a.Multiple sclerosis b.SLE c.Antiphospholipid syndrome d.Rheumatoid arthritis e.Myasthenia gravis

ANS: B, C, D, E Multiple sclerosis is not an autoimmune disorder. This patchy demyelination of the spinal cord may be a viral disorder. Autoimmune disorders (collagen vascular disease) make up a large group of conditions that disrupt the function of the immune system of the body. These disorders include those listed, as well as systemic sclerosis.

Because most pregnant women continue their usual activities, trauma remains a common complication during pregnancy. Approximately 1 in 12 pregnancies in the United States are complicated by trauma each year. As a result of the physiologic alterations that accompany pregnancy, special considerations for the mother and fetus are necessary when trauma occurs. Match the maternal system adaptation in pregnancy with the clinical response to trauma. a.Increased oxygen consumption b.Increased heart rate c.Decreased gastric motility d.Displacement of abdominal viscera e.Increase in clotting factors 1. Decreased placental perfusion in the supine position 2. Increased risk of thrombus formation 3. Altered pain referral 4. Increased risk of acidosis 5. Increased risk of aspiration

ANS: B, E, D, A, C NOT: Immediate priorities for the stabilization of the pregnant woman after trauma should be identical to that of the nonpregnant client after trauma. Fetal survival depends on maternal survival, and stabilization of the mother improves the chance of fetal well-being. Trauma may affect a number of systems within the body, and being aware of normal system alterations in the pregnant woman is important for the nurse who is caring for this client. Care should be adapted according to the body system that has been injured. The effects of trauma on pregnancy are also influenced by the length of gestation, type and severity of the injuries, and the degree of disruption of uterine and fetal physiologic features.

15. Which preexisting factor is known to increase the risk of GDM? a.Underweight before pregnancy b.Maternal age younger than 25 years c.Previous birth of large infant d.Previous diagnosis of type 2 diabetes mellitus

ANS: C A previous birth of a large infant suggests GDM. Obesity (body mass index [BMI] of 30 or greater) creates a higher risk for gestational diabetes. A woman younger than 25 years is not generally at risk for GDM. The person with type 2 diabetes mellitus already has diabetes and thus will continue to have it after pregnancy. Insulin may be required during pregnancy because oral hypoglycemia drugs are contraindicated during pregnancy.

15. A woman arrives at the emergency department with complaints of bleeding and cramping. The initial nursing history is significant for a last menstrual period 6 weeks ago. On sterile speculum examination, the primary care provider finds that the cervix is closed. The anticipated plan of care for this woman would be based on a probable diagnosis of which type of spontaneous abortion? a.Incomplete b.Inevitable c.Threatened d.Septic

ANS: C A woman with a threatened abortion has spotting, mild cramps, and no cervical dilation. A woman with an incomplete abortion would have heavy bleeding, mild-to-severe cramping, and cervical dilation. An inevitable abortion demonstrates the same symptoms as an incomplete abortion: heavy bleeding, mild-to-severe cramping, and cervical dilation. A woman with a septic abortion has malodorous bleeding and typically a dilated cervix. DIF: Cognitive Level: Understand REF: p. 670 TOP: Nursing Process: Planning MSC: Client Needs: Physiologic Integrity

3. A nurse is assessing a newborn girl who is 2 hours old. Which finding warrants a call to the health care provider? a. Blood glucose of 45 mg/dl using a Dextrostix screening method b. Heart rate of 160 beats per minute after vigorously crying c. Laceration of the cheek d. Passage of a dark black-green substance from the rectum

ANS: C Accidental lacerations can be inflicted by a scalpel during a cesarean birth. They are most often found on the scalp or buttocks and may require an adhesive strip for closure. Parents would be overly concerned about a laceration on the cheek. A blood glucose level of 45 mg/dl and a heart rate of 160 beats per minute after crying are both normal findings that do not warrant a call to the physician. The passage of meconium from the rectum is an expected finding in the newborn.

13. Which statement accurately describes an appropriate-for-gestational age (AGA) weight assessment? a. AGA weight assessment falls between the 25th and 75th percentiles for the infant's age. b. AGA weight assessment depends on the infant's length and the size of the newborn's head. c. AGA weight assessment falls between the 10th and 90th percentiles for the infant's age. d. AGA weight assessment is modified to consider intrauterine growth restriction (IUGR).

ANS: C An AGA weight falls between the 10th and 90th percentiles for the infant's age. The AGA range is larger than the 25th and 75th percentiles. The infant's length and head size are measured, but these measurements do not affect the normal weight designation. IUGR applies to the fetus, not to the newborn's weight.

1. An infant boy was delivered minutes ago. The nurse is conducting the initial assessment. Part of the assessment includes the Apgar score. When should the Apgar assessment be performed? a. Only if the newborn is in obvious distress b. Once by the obstetrician, just after the birth c. At least twice, 1 minute and 5 minutes after birth d. Every 15 minutes during the newborn's first hour after birth

ANS: C Apgar scoring is performed at 1 minute and at 5 minutes after birth. Scoring may continue at 5-minute intervals if the infant is in distress and requires resuscitation efforts. The Apgar score is performed on all newborns. Apgar score can be completed by the nurse or the birth attendant. The Apgar score permits a rapid assessment of the newborn's transition to extrauterine life. An interval of every 15 minutes is too long to wait to complete this assessment.

25. The nurse should be cognizant of which important information regarding the gastrointestinal (GI) system of the newborn? a. The newborn's cheeks are full because of normal fluid retention. b. The nipple of the bottle or breast must be placed well inside the baby's mouth because teeth have been developing in utero, and one or more may even be through. c. Regurgitation during the first day or two can be reduced by burping the infant and slightly elevating the baby's head. d. Bacteria are already present in the infant's GI tract at birth because they traveled through the placenta.

ANS: C Avoiding overfeeding can also reduce regurgitation. The newborn's cheeks are full because of well-developed sucking pads. Teeth do develop in utero, but the nipple is placed deep because the baby cannot move food from the lips to the pharynx. Bacteria are not present at birth, but they soon enter through various orifices.

20. What condition indicates concealed hemorrhage when the client experiences abruptio placentae? a.Decrease in abdominal pain b.Bradycardia c.Hard, boardlike abdomen d.Decrease in fundal height

ANS: C Concealed hemorrhage occurs when the edges of the placenta do not separate. The formation of a hematoma behind the placenta and subsequent infiltration of the blood into the uterine muscle results in a very firm, boardlike abdomen. Abdominal pain may increase. The client will have shock symptoms that include tachycardia. As bleeding occurs, the fundal height increases. DIF: Cognitive Level: Analyze REF: p. 683 TOP: Nursing Process: Assessment MSC: Client Needs: Physiologic Integrity

6. A woman arrives for evaluation of signs and symptoms that include a missed period, adnexal fullness, tenderness, and dark red vaginal bleeding. On examination, the nurse notices an ecchymotic blueness around the woman's umbilicus. What does this finding indicate? a.Normal integumentary changes associated with pregnancy b.Turner sign associated with appendicitis c.Cullen sign associated with a ruptured ectopic pregnancy d.Chadwick sign associated with early pregnancy

ANS: C Cullen sign, the blue ecchymosis observed in the umbilical area, indicates hematoperitoneum associated with an undiagnosed ruptured intraabdominal ectopic pregnancy. Linea nigra on the abdomen is the normal integumentary change associated with pregnancy and exhibits a brown pigmented, vertical line on the lower abdomen. Turner sign is ecchymosis in the flank area, often associated with pancreatitis. A Chadwick sign is a blue-purple cervix that may be seen during or around the eighth week of pregnancy. DIF: Cognitive Level: Analyze REF: p. 676 TOP: Nursing Process: Assessment MSC: Client Needs: Physiologic Integrity

2. During a prenatal visit, the nurse is explaining dietary management to a woman with pregestational diabetes. Which statement by the client reassures the nurse that teaching has been effective? a."I will need to eat 600 more calories per day because I am pregnant." b."I can continue with the same diet as before pregnancy as long as it is well balanced." c."Diet and insulin needs change during pregnancy." d."I will plan my diet based on the results of urine glucose testing."

ANS: C Diet and insulin needs change during the pregnancy in direct correlation to hormonal changes and energy needs. In the third trimester, insulin needs may double or even quadruple. The diet is individualized to allow for increased fetal and metabolic requirements, with consideration of such factors as prepregnancy weight and dietary habits, overall health, ethnic background, lifestyle, stage of pregnancy, knowledge of nutrition, and insulin therapy. Energy needs are usually calculated on the basis of 30 to 35 calories per kilogram of ideal body weight. Dietary management during a diabetic pregnancy must be based on blood, not urine, glucose changes.

6. Which important component of nutritional counseling should the nurse include in health teaching for a pregnant woman who is experiencing cholecystitis? a.Assess the woman's dietary history for adequate calories and proteins. b.Teach the woman that the bulk of calories should come from proteins. c.Instruct the woman to eat a low-fat diet and to avoid fried foods. d.Instruct the woman to eat a low-cholesterol, low-salt diet.

ANS: C Eating a low-fat diet and avoiding fried foods is appropriate nutritional counseling for this client. Caloric and protein intake do not predispose a woman to the development of cholecystitis. The woman should be instructed to limit protein intake and choose foods that are high in carbohydrates. A low-cholesterol diet may be the result of limiting fats. However, a low-salt diet is not indicated.

10. A woman at 28 weeks of gestation experiences blunt abdominal trauma as the result of a fall. The nurse must closely observe the client for what? a.Alteration in maternal vital signs, especially blood pressure b.Complaints of abdominal pain c.Placental absorption d.Hemorrhage

ANS: C Electronic fetal monitoring (EFM) tracings can help evaluate maternal status after trauma and can reflect fetal cardiac responses to hypoxia and hypoperfusion. Signs and symptoms of placental absorption include uterine irritability, contractions, vaginal bleeding, and changes in FHR characteristics. Hypoperfusion may be present in the pregnant woman before the onset of clinical signs of shock. EFM tracings show the first signs of maternal compromise, such as when the maternal heart rate, blood pressure, and color appear normal, yet the EFM printout shows signs of fetal hypoxia. Abdominal pain, in and of itself, is not the most important symptom. However, if it is accompanied by contractions, changes in the FHR, rupture of membranes, or vaginal bleeding, then the client should be evaluated for abruptio placentae. Clinical signs of hemorrhage do not appear until after a 30% loss of circulating volume occurs. Careful monitoring of fetal status significantly assists in maternal assessment, because the fetal monitor tracing works as an oximeter of internal well-being.

11. While assessing the integument of a 24-hour-old newborn, the nurse notes a pink papular rash with vesicles superimposed on the thorax, back, and abdomen. What action is the highest priority for the nurse to take at this time? a.Immediately notify the physician. b.Move the newborn to an isolation nursery. c.Document the finding as erythema toxicum neonatorum. d.Take the newborn's temperature, and obtain a culture of one of the vesicles.

ANS: C Erythema toxicum neonatorum (or erythema neonatorum) is a newborn rash that resembles flea bites. Notification of the physician, isolation of the newborn, or additional interventions are not necessary when erythema toxicum neonatorum is present.

22. A mother expresses fear about changing her infant's diaper after he is circumcised. What does the client need to be taught to care for her newborn son? a. Cleanse the penis with prepackaged diaper wipes every 3 to 4 hours. b. Apply constant, firm pressure by squeezing the penis with the fingers for at least 5 minutes if bleeding occurs. c. Gently cleanse the penis with water and apply petroleum jelly around the glans after each diaper change. d. Wash off the yellow exudate that forms on the glans at least once every day to prevent infection.

ANS: C Gently cleansing the penis with water and applying petroleum jelly around the glans after each diaper change are appropriate techniques when caring for an infant who has had a circumcision. With each diaper change, the penis should be washed with warm water to remove any urine or feces. If bleeding occurs, then the mother should apply gentle pressure to the site of the bleeding with a sterile gauze square. Yellow exudates are part of normal healing and cover the glans penis 24 hours after the circumcision; yellow exudates are not an infective process and should not be removed.

17. To manage her diabetes appropriately and to ensure a good fetal outcome, how would the pregnant woman with diabetes alter her diet? a.Eat six small equal meals per day. b.Reduce the carbohydrates in her diet. c.Eat her meals and snacks on a fixed schedule. d.Increase her consumption of protein.

ANS: C Having a fixed meal schedule will provide the woman and the fetus with a steady blood sugar level, provide a good balance with insulin administration, and help prevent complications. Having a fixed meal schedule is more important than the equal division of food intake. Approximately 45% of the food eaten should be in the form of carbohydrates.

10. Which statement regarding the laboratory test for glycosylated hemoglobin Alc is correct? a.The laboratory test for glycosylated hemoglobin Alc is performed for all pregnant women, not only those with or likely to have diabetes. b.This laboratory test is a snapshot of glucose control at the moment. c.This laboratory test measures the levels of hemoglobin Alc, which should remain at less than 7%. d.This laboratory test is performed on the woman's urine, not her blood.

ANS: C Hemoglobin Alc levels greater than 7% indicate an elevated glucose level during the previous 4 to 6 weeks. This extra laboratory test is for diabetic women and defines glycemic control over the previous 4 to 6 weeks. Glycosylated hemoglobin level tests are performed on the blood.

15. The nurse is teaching new parents about metabolic screening for the newborn. Which statement is most helpful to these clients? a. All states test for phenylketonuria (PKU), hypothyroidism, cystic fibrosis, and sickle cell diseases. b. Federal law prohibits newborn genetic testing without parental consent. c. If genetic screening is performed before the infant is 24 hours old, then it should be repeated at age 1 to 2 weeks. d. Hearing screening is now mandated by federal law.

ANS: C If testing is performed before the infant is 24 hours old, then genetic screening should be repeated when the infant is 1 to 2 weeks old. All states test for PKU and hypothyroidism but not for other genetic defects. Federal law mandates newborn genetic screening; however, parents can decline the testing. A waiver should be signed, and a notation made in the infant's medical record. Federal law does not mandate screening for hearing problems; however, the majority of states have enacted legislation mandating newborn hearing screening. In the United States, the majority (95%) of infants are screened for hearing loss before discharge from the hospital.

16. A pregnant woman at term is transported to the emergency department (ED) after a severe vehicular accident. The obstetric nurse responds and rushes to the ED with a fetal monitor. Cardiopulmonary arrest occurs as the obstetric nurse arrives. What is the highest priority for the trauma team? a.Obtaining IV access, and starting aggressive fluid resuscitation b.Quickly applying the fetal monitor to determine whether the fetus viability c.Starting cardiopulmonary resuscitation (CPR) d.Transferring the woman to the surgical unit for an emergency cesarean delivery in case the fetus is still alive

ANS: C In a situation of severe maternal trauma, the systematic evaluation begins with a primary survey and the initial ABCs (airway, breathing, and circulation) of resuscitation. CPR is initiated first, followed by intravenous (IV) replacement fluid. After immediate resuscitation and successful stabilization measures, a more detailed secondary survey of the mother and fetus should be accomplished. Attempts at maternal resuscitation are made, followed by a secondary survey of the fetus. In the presence of multisystem trauma, a cesarean delivery may be indicated to increase the chance for maternal survival.

33. How would the nurse differentiate a meconium stool from a transitional stool in the healthy newborn? a. Observed at age 3 days b. Is residue of a milk curd c. Passes in the first 12 hours of life d. Is lighter in color and looser in consistency

ANS: C Meconium stool is usually passed in the first 12 hours of life, and 99% of newborns have their first stool within 48 hours. If meconium is not passed by 48 hours, then obstruction is suspected. Meconium stool is the first stool of the newborn and is made up of matter remaining in the intestines during intrauterine life. Meconium is dark and sticky

3. The nurse is preparing to administer methotrexate to the client. This hazardous drug is mostoften used for which obstetric complication? a.Complete hydatidiform mole b.Missed abortion c.Unruptured ectopic pregnancy d.Abruptio placentae

ANS: C Methotrexate is an effective nonsurgical treatment option for a hemodynamically stable woman whose ectopic pregnancy is unruptured and measures less than 4 cm in diameter. Methotrexate is not indicated or recommended as a treatment option for a complete hydatidiform mole, for a missed abortion, or for abruptio placentae. DIF: Cognitive Level: Apply REF: p. 677 TOP: Nursing Process: Planning MSC: Client Needs: Physiologic Integrity

10. The management of the pregnant client who has experienced a pregnancy loss depends on the type of miscarriage and the signs and symptoms. While planning care for a client who desires outpatient management after a first-trimester loss, what would the nurse expect the plan to include? a.Dilation and curettage (D&C) b.Dilation and evacuation (D&E) c.Misoprostol d.Ergot products

ANS: C Outpatient management of a first-trimester loss is safely accomplished by the intravaginal use of misoprostol for up to 2 days. If the bleeding is uncontrollable, vital signs are unstable, or signs of infection are present, then a surgical evacuation should be performed. D&C is a surgical procedure that requires dilation of the cervix and scraping of the uterine walls to remove the contents of pregnancy. This procedure is commonly performed to treat inevitable or incomplete abortion and should be performed in a hospital. D&E is usually performed after 16 weeks of pregnancy. The cervix is widely dilated, followed by removal of the contents of the uterus. Ergot products such as Methergine or Hemabate may be administered for excessive bleeding after miscarriage. DIF: Cognitive Level: Apply REF: p. 672 TOP: Nursing Process: Planning MSC: Client Needs: Physiologic Integrity

20. What marks on a baby's skin may indicate an underlying problem that requires notification of a physician? a. Mongolian spots on the back b. Telangiectatic nevi on the nose or nape of the neck c. Petechiae scattered over the infant's body d. Erythema toxicum neonatorum anywhere on the body

ANS: C Petechiae (bruises) scattered over the infant's body should be reported to the pediatrician because they may indicate underlying problems. Mongolian spots are bluish-black spots that resemble bruises but gradually fade over months and have no clinical significance. Telangiectatic nevi (stork bites, angel kisses) fade by the second year and have no clinical significance. Erythema toxicum neonatorum is an appalling-looking rash; however, it has no clinical significance and requires no treatment.

29. A first-time dad is concerned that his 3-day-old daughter's skin looks "yellow." In the nurse's explanation of physiologic jaundice, what fact should be included? a. Physiologic jaundice occurs during the first 24 hours of life. b. Physiologic jaundice is caused by blood incompatibilities between the mother and the infant blood types. c. Physiologic jaundice becomes visible when serum bilirubin levels peak between the second and fourth days of life. d. Physiologic jaundice is also known as breast milk jaundice.

ANS: C Physiologic jaundice becomes visible when the serum bilirubin reaches a level of 5 mg/dl or higher when the baby is approximately 3 days old. This finding is within normal limits for the newborn. Pathologic jaundice, not physiologic jaundice, occurs during the first 24 hours of life and is caused by blood incompatibilities that result in excessive destruction of erythrocytes; this condition must be investigated. Breast milk jaundice occurs in one third of breastfed infants at 2 weeks and is caused by an insufficient intake of fluids.

23. Which finding on a prenatal visit at 10 weeks of gestation might suggest a hydatidiform mole? a.Complaint of frequent mild nausea b.Blood pressure of 120/80 mm Hg c.Fundal height measurement of 18 cm d.History of bright red spotting for 1 day, weeks ago

ANS: C The uterus in a hydatidiform molar pregnancy is often larger than would be expected on the basis of the duration of the pregnancy. Nausea increases in a molar pregnancy because of the increased production of hCG. A woman with a molar pregnancy may have early-onset pregnancy-induced hypertension. In the client's history, bleeding is normally described as brownish. DIF: Cognitive Level: Analyze REF: p. 678 TOP: Nursing Process: Assessment MSC: Client Needs: Health Promotion and Maintenance

8. A number of metabolic changes occur throughout pregnancy. Which physiologic adaptation of pregnancy will influence the nurse's plan of care? a.Insulin crosses the placenta to the fetus only in the first trimester, after which the fetus secretes its own. b.Women with insulin-dependent diabetes are prone to hyperglycemia during the first trimester because they are consuming more sugar. c.During the second and third trimesters, pregnancy exerts a diabetogenic effect that ensures an abundant supply of glucose for the fetus. d.Maternal insulin requirements steadily decline during pregnancy.

ANS: C Pregnant women develop increased insulin resistance during the second and third trimesters. Insulin never crosses the placenta; the fetus starts making its own around the 10th week. As a result of normal metabolic changes during pregnancy, insulin-dependent women are prone to hypoglycemia (low levels). Maternal insulin requirements may double or quadruple by the end of pregnancy.

5. A woman with asthma is experiencing a postpartum hemorrhage. Which drug should be avoided when treating postpartum bleeding to avoid exacerbating asthma? a.Oxytocin (Pitocin) b.Nonsteroidal antiinflammatory drugs (NSAIDs) c.Hemabate d.Fentanyl

ANS: C Prostaglandin derivatives should not be used to treat women with asthma, because they may exacerbate symptoms. Oxytocin is the drug of choice to treat this woman's bleeding; it will not exacerbate her asthma. NSAIDs are not used to treat bleeding. Fentanyl is used to treat pain, not bleeding.

8. A woman who is 30 weeks of gestation arrives at the hospital with bleeding. Which differential diagnosis would not be applicable for this client? a.Placenta previa b.Abruptio placentae c.Spontaneous abortion d.Cord insertion

ANS: C Spontaneous abortion is another name for miscarriage it occurs, by definition, early in pregnancy. Placenta previa is a well-known reason for bleeding late in pregnancy. The premature separation of the placenta (abruptio placentae) is a bleeding disorder that can occur late in pregnancy. Cord insertion may cause a bleeding disorder that can also occur late in pregnancy. DIF: Cognitive Level: Understand REF: p. 669 TOP: Nursing Process: Assessment MSC: Client Needs: Physiologic Integrity, Physiologic Adaptation

6. While examining a newborn, the nurse notes uneven skinfolds on the buttocks and a clunk when performing the Ortolani maneuver. These findings are likely indicative of what? a.Polydactyly b.Clubfoot c.Hip dysplasia d.Webbing

ANS: C The Ortolani maneuver is used to detect the presence of hip dysplasia. Polydactyly is the presence of extra digits. Clubfoot (talipes equinovarus) is a deformity in which the foot turns inward and is fixed in a plantar-flexion position. Webbing, or syndactyly, is a fusing of the fingers or toes.

3. The nurse is assessing a full term, quiet, and alert newborn. What is the average expected apical pulse range (in beats per minute)? a.80 to 100 b.100 to 120 c.120 to 160 d.150 to 180

ANS: C The average infant heart rate while awake is 120 to 160 beats per minute. The newborn's heart rate may be approximately 85 to 100 beats per minute while sleeping and typically a little higher than 100 to 120 beats per minute when alert but quiet. A heart rate of 150 to 180 beats per minute is typical when the infant cries.

32. The nurse is cognizant of which information related to the administration of vitamin K? a. Vitamin K is important in the production of red blood cells. b. Vitamin K is necessary in the production of platelets. c. Vitamin K is not initially synthesized because of a sterile bowel at birth. d. Vitamin K is responsible for the breakdown of bilirubin and the prevention of jaundice.

ANS: C The bowel is initially sterile in the newborn, and vitamin K cannot be synthesized until food is introduced into the bowel. Vitamin K is necessary to activate blood-clotting factors. The platelet count in term newborns is near adult levels. Vitamin K is necessary to activate prothrombin and other blood-clotting factors.

6. A newborn is jaundiced and is receiving phototherapy via ultraviolet bank lights. What is themost appropriate nursing intervention when caring for an infant with hyperbilirubinemia and receiving phototherapy? a. Applying an oil-based lotion to the newborn's skin to prevent dying and cracking b. Limiting the newborn's intake of milk to prevent nausea, vomiting, and diarrhea c. Placing eye shields over the newborn's closed eyes d. Changing the newborn's position every 4 hours

ANS: C The infant's eyes must be protected by an opaque mask to prevent overexposure to the light. Eye shields should completely cover the eyes but not occlude the nares. Lotions and ointments should not be applied to the infant because they absorb heat and can cause burns. The lights increase insensible water loss, placing the infant at risk for fluid loss and dehydration. Therefore, adequate hydration is important for the infant. The infant should be turned every 2 hours to expose all body surfaces to the light.

18. If the newborn has excess secretions, the mouth and nasal passages can be easily cleared with a bulb syringe. How should the nurse instruct the parents on the use of this instrument? a. Avoid suctioning the nares. b. Insert the compressed bulb into the center of the mouth. c. Suction the mouth first. d. Remove the bulb syringe from the crib when finished.

ANS: C The mouth should always be suctioned first to prevent the infant from inhaling pharyngeal secretions by gasping as the nares are suctioned. After compressing the bulb, the syringe should be inserted into one side of the mouth. If it is inserted into the center of the mouth, then the gag reflex is likely to be initiated. When the infant's cry no longer sounds as though it is through mucus or a bubble, suctioning can be stopped. The nasal passages should be suctioned one nostril at a time.The bulb syringe should remain in the crib so that it is easily accessible if needed again.

4. A newborn is placed under a radiant heat warmer. The nurse understands that thermoregulation presents a problem for the newborn. What is the rationale for this difficulty? a. The renal function of a newborn is not fully developed, and heat is lost in the urine. b. The small body surface area of a newborn favors more rapid heat loss than does an adult's body surface area. c. Newborns have a relatively thin layer of subcutaneous fat that provides poor insulation. d. Their normal flexed posture favors heat loss through perspiration.

ANS: C The newborn has little thermal insulation. Furthermore, the blood vessels are closer to the surface of the skin. Changes in environmental temperature alter the temperature of the blood, thereby influencing temperature regulation centers in the hypothalamus. Heat loss does not occur through urination. Newborns have a higher body surface-to-weight ratio than adults. The flexed position of the newborn helps guard against heat loss, because it diminishes the amount of body surface exposed to the environment.

15. Which information related to the newborn's developing cardiovascular system should the nurse fully comprehend? a. The heart rate of a crying infant may rise to 120 beats per minute. b. Heart murmurs heard after the first few hours are a cause for concern. c. The point of maximal impulse (PMI) is often visible on the chest wall. d. Persistent bradycardia may indicate respiratory distress syndrome (RDS).

ANS: C The newborn's thin chest wall often allows the PMI to be observed. The normal heart rate for infants who are not sleeping is 120 to 160 beats per minute. However, a crying infant could temporarily have a heart rate of 180 beats per minute. Heart murmurs during the first few days of life have no pathologic significance; however, an irregular heart rate beyond the first few hours should be further evaluated. Persistent tachycardia may indicate RDS; bradycardia may be a sign of congenital heart blockage.

4. A woman has experienced iron deficiency anemia during her pregnancy. She had been taking iron for 3 months before the birth. The client gave birth by cesarean 2 days earlier and has been having problems with constipation. After assisting her back to bed from the bathroom, the nurse notes that the woman's stools are dark (greenish-black). What should the nurse's initial action be? a.Perform a guaiac test, and record the results. b.Recognize the finding as abnormal, and report it to the primary health care provider. c.Recognize the finding as a normal result of iron therapy. d.Check the woman's next stool to validate the observation.

ANS: C The nurse should recognize that dark stools are a common side effect in clients who are taking iron replacement therapy. A guaiac test would be indicated if gastrointestinal (GI) bleeding was suspected. GI irritation, including dark stools, is also a common side effect of iron therapy. Observation of stool formation is a normal nursing activity.

9. The nurse is performing a gestational age and physical assessment on the newborn. The infant appears to have an excessive amount of saliva. This clinical finding may be indicative of what? a. Excessive saliva is a normal finding in the newborn. b. Excessive saliva in a neonate indicates that the infant is hungry. c. It may indicate that the infant has a tracheoesophageal fistula or esophageal atresia. d. Excessive saliva may indicate that the infant has a diaphragmatic hernia.

ANS: C The presence of excessive saliva in a neonate should alert the nurse to the possibility of a tracheoesophageal fistula or esophageal atresia. Excessive salivation may not be a normal finding and should be further assessed for the possibility that the infant has an esophageal abnormality. The hungry infant reacts by making sucking motions, rooting, or making hand-to-mouth movements. The infant with a diaphragmatic hernia exhibits severe respiratory distress.

4. What is the rationale for the administration of vitamin K to the healthy full-term newborn? a. Most mothers have a diet deficient in vitamin K, which results in the infant being deficient. b. Vitamin K prevents the synthesis of prothrombin in the liver and must be administered by injection. c. Bacteria that synthesize vitamin K are not present in the newborn's intestinal tract. d. The supply of vitamin K in the healthy full-term newborn is inadequate for at least 3 to 4 months and must be supplemented.

ANS: C Vitamin K is provided because the newborn does not have the intestinal flora to produce this vitamin for the first week. The maternal diet has no bearing on the amount of vitamin K found in the newborn. Vitamin K promotes the formation of clotting factors in the liver and is used for the prevention and treatment of hemorrhagic disease in the newborn. Vitamin K is not produced in the intestinal tract of the newborn until after microorganisms are introduced. By day 8, normal newborns are able to produce their own vitamin K.

11. What is the primary rationale for nurses wearing gloves when handling the newborn? a. To protect the baby from infection b. As part of the Apgar protocol c. To protect the nurse from contamination by the newborn d. Because the nurse has the primary responsibility for the baby during the first 2 hours

ANS: C With the possibility of transmission of viruses such as HBV and the human immunodeficiency virus (HIV) through maternal blood and amniotic fluid, the newborn must be considered a potential contamination source until proven otherwise. As part of Standard Precautions, nurses should wear gloves when handling the newborn until blood and amniotic fluid are removed by bathing. Proper hand hygiene is all that is necessary to protect the infant from infection. Wearing gloves is not necessary to complete the Apgar score assessment. The nurse assigned to the mother-baby couplet has primary responsibility for the newborn, regardless of whether or not she wears gloves.

11. Which neurologic condition would require preconception counseling, if at all possible? a.Eclampsia b.Bell palsy c.Epilepsy d.Multiple sclerosis

ANS: C Women with epilepsy should receive preconception counseling, if at all possible. Achieving seizure control before becoming pregnant is a desirable state. Medication should also be carefully reviewed. Eclampsia may sometimes be confused with epilepsy, and Bell palsy is a form of facial paralysis; preconception counseling for either condition is not essential to care. Multiple sclerosis is a patchy demyelination of the spinal cord that does not affect the normal course of pregnancy or birth.

2. Hypothyroidism occurs in 2 to 3 pregnancies per 1000. Because severe hypothyroidism is associated with infertility and miscarriage, it is not often seen in pregnancy. Regardless of this fact, the nurse should be aware of the characteristic symptoms of hypothyroidism. Which do they include? (Select all that apply.) a.Hot flashes b.Weight loss c.Lethargy d.Decrease in exercise capacity e.Cold intolerance

ANS: C, D, E Symptoms include weight gain, lethargy, decrease in exercise capacity, and intolerance to cold. Other presentations might include constipation, hoarseness, hair loss, and dry skin. Thyroid supplements are used to treat hyperthyroidism in pregnancy

13. A woman with gestational diabetes has had little or no experience reading and interpreting glucose levels. The client shows the nurse her readings for the past few days. Which reading signals the nurse that the client may require an adjustment of insulin or carbohydrates? a.75 mg/dl before lunch. This is low; better eat now. b.115 mg/dl 1 hour after lunch. This is a little high; maybe eat a little less next time. c.115 mg/dl 2 hours after lunch. This is too high; it is time for insulin. d.50 mg/dl just after waking up from a nap. This is too low; maybe eat a snack before going to sleep

ANS: D 50 mg/dl after waking from a nap is too low. During hours of sleep, glucose levels should not be less than 60 mg/dl. Snacks before sleeping can be helpful. The premeal acceptable range is 60 to 99 mg/dl. The readings 1 hour after a meal should be less than 129 mg/dl. Two hours after eating, the readings should be less than 120 mg/dl.

5. An African-American woman noticed some bruises on her newborn daughter's buttocks. The client asks the nurse what causes these. How would the nurse best explain this integumentary finding to the client? a.Lanugo b.Vascular nevus c.Nevus flammeus d.Mongolian spot

ANS: D A Mongolian spot is a bluish-black area of pigmentation that may appear over any part of the exterior surface of the infant's body and is more commonly noted on the back and buttocks and most frequently observed on infants whose ethnic origins are Mediterranean, Latin American, Asian, or African. Lanugo is the fine, downy hair observed on a term newborn. A vascular nevus, commonly called a strawberry mark, is a type of capillary hemangioma. A nevus flammeus, commonly called a port wine stain, is most frequently found on the face.

19. As part of the infant discharge instructions, the nurse is reviewing the use of the infant car safety seat. Which information is the highest priority for the nurse to share? a. Infant carriers are okay to use until an infant car safety seat can be purchased. b. For traveling on airplanes, buses, and trains, infant carriers are satisfactory. c. Infant car safety seats are used for infants only from birth to 15 pounds. d. Infant car seats should be rear facing and placed in the back seat of the car.

ANS: D An infant placed in the front seat could be severely injured by an air bag that deploys during an automobile accident. Infants should travel only in federally approved, rear-facing safety seats secured in the rear seat and only in federally approved safety seats even when traveling on a commercial vehicle. Infants should use a rear-facing car seat from birth to 20 pounds and to age 1 year.

24. How should the nurse interpret an Apgar score of 10 at 1 minute after birth? a. The infant is having no difficulty adjusting to extrauterine life and needs no further testing. b. The infant is in severe distress and needs resuscitation. c. The nurse predicts a future free of neurologic problems. d. The infant is having no difficulty adjusting to extrauterine life but should be assessed again at 5 minutes after birth.

ANS: D An initial Apgar score of 10 is a good sign of healthy adaptation; however, the test must be repeated at the 5-minute mark.

7. The nurse who elects to practice in the area of women's health must have a thorough understanding of miscarriage. Which statement regarding this condition is most accurate? a.A miscarriage is a natural pregnancy loss before labor begins. b.It occurs in fewer than 5% of all clinically recognized pregnancies. c.Careless maternal behavior, such as poor nutrition or excessive exercise, can be a factor in causing a miscarriage. d.If a miscarriage occurs before the 12th week of pregnancy, then it may be observed only as moderate discomfort and blood loss.

ANS: D Before the sixth week, the only evidence might be a heavy menstrual flow. After the 12th week, more severe pain, similar to that of labor, is likely. Miscarriage is a natural pregnancy loss, but it occurs, by definition, before 20 weeks of gestation, before the fetus is viable. Miscarriages occur in approximately 10% to 15% of all clinically recognized pregnancies. Miscarriages can be caused by a number of disorders or illnesses outside the mother's control or knowledge. DIF: Cognitive Level: Understand REF: p. 670 TOP: Nursing Process: Assessment MSC: Client Needs: Physiologic Integrity

7. Postoperative care of the pregnant woman who requires abdominal surgery for appendicitis includes which additional assessment? a.Intake and output (I&O) and intravenous (IV) site b.Signs and symptoms of infection c.Vital signs and incision d.Fetal heart rate (FHR) and uterine activity

ANS: D Care of a pregnant woman undergoing surgery for appendicitis differs from that for a nonpregnant woman in one significant aspect: the presence of the fetus. Continuous fetal and uterine monitoring should take place. An assessment of I&O levels, along with an assessment of the IV site, are normal postoperative care procedures. Evaluating the client for signs and symptoms of infection is also part of routine postoperative care. Routine vital signs and evaluation of the incision site are expected components of postoperative care.

8. Since the gene for cystic fibrosis was identified in 1989, data can be collected for the purposes of genetic counseling for couples regarding carrier status. According to the most recent statistics, how often does cystic fibrosis occur in Caucasian live births? a.1 in 100 b.1 in 1000 c.1 in 2000 d.1 in 3200

ANS: D Cystic fibrosis occurs in approximately 1 in 3200 Caucasian live births. 1 in 100, 1 in 1000, and 1 in 2000 occurrences of cystic fibrosis in live births are all too frequent rates.

13. What form of heart disease in women of childbearing years generally has a benign effect on pregnancy? a.Cardiomyopathy b.Rheumatic heart disease c.Congenital heart disease d.Mitral valve prolapse

ANS: D Mitral valve prolapse is a benign condition that is usually asymptomatic. Cardiomyopathy produces congestive heart failure during pregnancy. Rheumatic heart disease can lead to heart failure during pregnancy. Some congenital heart diseases produce pulmonary hypertension or endocarditis during pregnancy.

27. A nursing student is helping the nursery nurses with morning vital signs. A baby born 10 hours ago by cesarean section is found to have moist lung sounds. What is the best interpretation of these data? a. The nurse should immediately notify the pediatrician for this emergency situation. b. The neonate must have aspirated surfactant. c. If this baby was born vaginally, then a pneumothorax could be indicated. d. The lungs of a baby delivered by cesarean section may sound moist during the first 24 hours after childbirth.

ANS: D Moist lung sounds will resolve within a few hours. A surfactant acts to keep the expanded alveoli partially open between respirations for this common condition of newborns. In a vaginal birth, absorption of the remaining lung fluid is accelerated by the process of labor and delivery. The remaining lung fluid will move into interstitial spaces and be absorbed by the circulatory and lymphatic systems. Moist lung sounds are particularly common in infants delivered by cesarean section. The surfactant is produced by the lungs; therefore, aspiration is not a concern.

14. A pregnant woman at 33 weeks of gestation is brought to the birthing unit after a minor automobile accident. The client is experiencing no pain and no vaginal bleeding, her vital signs are stable, and the FHR is 132 beats per minute with variability. What is the nurse's highestpriority? a.Monitoring the woman for a ruptured spleen b.Obtaining a physician's order to discharge her home c.Monitoring her for 24 hours d.Using continuous EFM for a minimum of 4 hours

ANS: D Monitoring the external FHR and contractions is recommended after blunt trauma in a viable gestation for a minimum of 4 hours, regardless of injury severity. Fetal monitoring should be initiated as soon as the woman is stable. In this scenario, no clinical findings indicate the possibility of a ruptured spleen. If the maternal and fetal findings are normal, then EFM should continue for a minimum of 4 hours after a minor trauma or a minor automobile accident. Once the monitoring has been completed and the health care provider is reassured of fetal well-being, the client may be discharged home. Monitoring for 24 hours is unnecessary unless the ERM strip is abnormal or nonreassuring.

17. Another common pregnancy-specific condition is pruritic urticarial papules and plaques of pregnancy (PUPPP). A client asks the nurse why she has developed this condition and what can be done. What is the nurse's bestresponse? a.PUPPP is associated with decreased maternal weight gain. b.The rate of hypertension decreases with PUPPP. c.This common pregnancy-specific condition is associated with a poor fetal outcome. d.The goal of therapy is to relieve discomfort.

ANS: D PUPPP is associated with increased maternal weight gain, increased rate of twin gestation, and hypertension. It is not, however, associated with poor maternal or fetal outcomes. The goal of therapy is simply to relieve discomfort. Antipruritic topical medications, topical steroids, and antihistamines usually provide relief. PUPPP usually resolves before childbirth or shortly thereafter.

16. Which physiologic alteration of pregnancy most significantly affects glucose metabolism? a.Pancreatic function in the islets of Langerhans is affected by pregnancy. b.Pregnant women use glucose at a more rapid rate than nonpregnant women. c.Pregnant women significantly increase their dietary intake. d.Placental hormones are antagonistic to insulin, thus resulting in insulin resistance.

ANS: D Placental hormones, estrogen, progesterone, and human placental lactogen (HPL) create insulin resistance. Insulin is also broken down more quickly by the enzyme placental insulinase. Pancreatic functioning is not affected by pregnancy. The glucose requirements differ because of the growing fetus. The pregnant woman should increase her intake by 200 calories a day.

29. Screening for critical congenital heart disease (CCHD) was added to the uniform screening panel in 2011. The nurse has explained this testing to the new mother. Which action by the nurse related to this test is correct? a. Screening is performed when the infant is 12 hours of age. b. Testing is performed with an electrocardiogram. c. Oxygen (O2) is measured in both hands and in the right foot. d. A passing result is an O2 saturation of ≥95%.

ANS: D Screening is performed when the infant is between 24 and 48 hours of age. The test is performed using pulse oximetry technology. O2 is measured in the right hand and one foot. A passing result is an O2 saturation of ≥95% with a ≤3% absolute difference between upper and lower extremity readings.

24. A 32-year-old primigravida is admitted with a diagnosis of ectopic pregnancy. Which information assists the nurse in developing the plan of care? a.Bed rest and analgesics are the recommended treatment. b.She will be unable to conceive in the future. c.A D&C will be performed to remove the products of conception. d.Hemorrhage is the primary concern.

ANS: D Severe bleeding occurs if the fallopian tube ruptures. The recommended treatment is to remove the pregnancy before rupture to prevent hemorrhaging. If the tube must be removed, then the woman's fertility will decrease however, she will not be infertile. A D&C is performed on the inside of the uterine cavity. The ectopic pregnancy is located within the tubes. DIF: Cognitive Level: Apply REF: p. 676 TOP: Nursing Process: Planning MSC: Client Needs: Physiologic Integrity

1. When caring for a pregnant woman with cardiac problems, the nurse must be alert for the signs and symptoms of cardiac decompensation. Which critical findings would the nurse find on assessment of the client experiencing this condition? a.Regular heart rate and hypertension b.Increased urinary output, tachycardia, and dry cough c.Shortness of breath, bradycardia, and hypertension d.Dyspnea, crackles, and an irregular, weak pulse

ANS: D Signs of cardiac decompensation include dyspnea; crackles; an irregular, weak, and rapid pulse; rapid respirations; a moist and frequent cough; generalized edema; increasing fatigue; and cyanosis of the lips and nailbeds. A regular heart rate and hypertension are not generally associated with cardiac decompensation. Of the symptoms of increased urinary output, tachycardia, and dry cough, only tachycardia is indicative of cardiac decompensation. Of the symptoms of shortness of breath, bradycardia, and hypertension, only dyspnea is indicative of cardiac decompensation.

24. While evaluating the reflexes of a newborn, the nurse notes that with a loud noise the newborn symmetrically abducts and extends his arms, his fingers fan out and form a C with the thumb and forefinger, and he has a slight tremor. The nurse would document this finding as a positive _____ reflex. a. tonic neck b. glabellar (Myerson) c. Babinski d. Moro

ANS: D The characteristics displayed by the infant are associated with a positive Moro reflex. The tonic neck reflex occurs when the infant extends the leg on the side to which the infant's head simultaneously turns. The glabellar (Myerson) reflex is elicited by tapping on the infant's head while the eyes are open. A characteristic response is blinking for the first few taps. The Babinski reflex occurs when the sole of the foot is stroked upward along the lateral aspect of the sole and then across the ball of the foot. A positive response occurs when all the toes hyperextend, with dorsiflexion of the big toe.

26. As part of their teaching function at discharge, nurses should educate parents regarding safe sleep. Based on the most recent evidence, which information is incorrect and should be discussed with parents? a. Prevent exposure to people with upper respiratory tract infections. b. Keep the infant away from secondhand smoke. c. Avoid loose bedding, water beds, and beanbag chairs. d. Place the infant on his or her abdomen to sleep.

ANS: D The infant should be laid down to sleep on his or her back for better breathing and to prevent sudden infant death syndrome (SIDS). Grandmothers may encourage the new parents to place the infant on the abdomen; however, evidence shows "back to sleep" reduces SIDS. Infants are vulnerable to respiratory infections; therefore, infected people must be kept away. Secondhand smoke can damage lungs. Infants can suffocate in loose bedding and in furniture that can trap them. Per AAP guidelines, infants should always be placed "back to sleep" and allowed tummy time to play to prevent plagiocephaly.

7. Early this morning, an infant boy was circumcised using the PlastiBell method. Based on the nurse's evaluation, when will the infant be ready for discharge? a. When the bleeding completely stops b. When yellow exudate forms over the glans c. When the PlastiBell plastic rim (bell) falls off d. When the infant voids

ANS: D The infant should be observed for urination after the circumcision. Bleeding is a common complication after circumcision, and the nurse will check the penis for 12 hours after a circumcision to assess and provide appropriate interventions for the prevention and treatment of bleeding. Yellow exudate covers the glans penis in 24 hours after the circumcision and is part of normal healing; yellow exudate is not an infective process. The PlastiBell plastic rim (bell) remains in place for approximately a week and falls off when healing has taken place.

8. What is the most critical physiologic change required of the newborn after birth? a.Closure of fetal shunts in the circulatory system b.Full function of the immune defense system c.Maintenance of a stable temperature d.Initiation and maintenance of respirations

ANS: D The most critical adjustment of a newborn at birth is the establishment of respirations. The cardiovascular system changes significantly after birth as a result of fetal respirations, which reduce pulmonary vascular resistance to the pulmonary blood flow and initiate a chain of cardiac changes that support the cardiovascular system. After the establishment of respirations, heat regulation is critical to newborn survival. The infant relies on passive immunity received from the mother for the first 3 months of life.

17. Which infant response to cool environmental conditions is either not effective or not available to them? a. Constriction of peripheral blood vessels b. Metabolism of brown fat c. Increased respiratory rates d. Unflexing from the normal position

ANS: D The newborn's flexed position guards against heat loss, because it reduces the amount of body surface exposed to the environment. The newborn's body is able to constrict the peripheral blood vessels to reduce heat loss. Burning brown fat generates heat. The respiratory rate may rise to stimulate muscular activity, which generates heat.

20. A nurse is responsible for teaching new parents regarding the hygienic care of their newborn. Which instruction should the nurse provide regarding bathing? a. Avoid washing the head for at least 1 week to prevent heat loss. b. Sponge bathe the newborn for the first month of life. c. Cleanse the ears and nose with cotton-tipped swabs, such as Q-tips. d. Create a draft-free environment of at least 24° C (75° F) when bathing the infant.

ANS: D The temperature of the room should be 24° C (75° F), and the bathing area should be free of drafts. To prevent heat loss, the infant's head should be bathed before unwrapping and undressing. Tub baths may be initiated from birth. Ensure that the infant is fully immersed. Q-tips should not be used; they may cause injury. A corner of a moistened washcloth should be twisted into shape so that it can be used to cleanse the ears and nose.

21. The brain is vulnerable to nutritional deficiencies and trauma in early infancy. What is the rationale for this physiologic adaptation in the newborn? a. Incompletely developed neuromuscular system b. Primitive reflex system c. Presence of various sleep-wake states d. Cerebellum growth spurt

ANS: D The vulnerability of the brain is likely due to the cerebellum growth spurt. By the end of the first year, the cerebellum ends its growth spurt that began at approximately 30 weeks of gestation. The neuromuscular system is almost completely developed at birth. The reflex system is not relevant to the cerebellum growth spurt. The various sleep-wake states are not relevant to the cerebellum growth spurt.

MATCHING The healthy infant must accomplish both behavioral and biologic tasks to develop normally. Behavioral characteristics form the basis of the social capabilities of the infant. Newborns pass through a hierarchy of developmental challenges as they adapt to their environment and caregivers. This progression in behavior is the basis for the Brazelton Neonatal Behavioral Assessment (NBAS). Match the cluster of neonatal behaviors with the correct level on the NBAS scale. a. Habituation b. Orientation c. Range of state d. Autonomic stability e.Regulation of state 1. Signs of stress related to homeostatic adjustment 2. Ability to respond to discrete stimuli while asleep 3. Measure of general arousability 4. How the infant responds when aroused 5. Ability to attend to visual and auditory stimuli while alert

ANS: D, A, C, E, B


Set pelajaran terkait

Modern Database Management Chapter 9 - Data Warehousing

View Set

PS121 Exam #2- Presidency & Bureaucracy

View Set

Anatomy: Chapter 23 Digestive System

View Set

INTRODUCTION TO BUSINESS & MANAGEMENT

View Set

Foundations CH 37 loss and grief

View Set

NCLEX mental health questions for exam 2

View Set

Management Final Exam; ch. 3, 11, 12, 13, 14, 15, 16

View Set

Culture & Diversity NCLEX Questions

View Set

Physics Final Unit 1: Laws of Motion & Mechanics

View Set